You are on page 1of 91

Path Exam r ecalls fr om 2003-2005 - Questions

originally collated by DR, JD et al. Re-arranged by TJP

PATH EXAM RECALLS FROM 2003-2005 - QUESTIONS .............................................................1 CHEST...........................................................................................................................................2 CNS .............................................................................................................................................20 ENDO...........................................................................................................................................32 ABDO...........................................................................................................................................35 HEPATOBILIARY AND PANCREAS.....................................................................................................43 HAEM...........................................................................................................................................44 MSK .............................................................................................................................................47 PAEDIATRICS .................................................................................................................................53 BREAST.......................................................................................................................................61 MISC ............................................................................................................................................65 GU................................................................................................................................................69 HEAD AND NECK ............................................................................................................................74 O&G .............................................................................................................................................77 CVS..............................................................................................................................................81

Page 1

Explanation of document Dinesh and Julies files covering recalls from 2003-2005, collated, renumbered, answers both from their study groups answers and looking it all up in Robbins. Formatting: Answers are in another style CorrectAnswer. Change this to a black color and unbold to print without answers. Occasional extra text in HiddenExplanation. Make this white to hide when printing.

CHEST
2005 APRIL PATHOLOGY RECALLED MCQS 1. Pathologist is looking for a gross specimen of adenocarcinoma of the lung to show medical students. He has a specimen of SCC only. RE: Adenocarinoma vs SCC lung a) similar macroscopically b) adeno more common in female, less associated with smoking 2. Staging Ca lung mass invading mediastinum with mediastinal and hilar nodes. No distant mets. What else is necessary for further staging? a) Size of mediastinal nodes b) Size of mass c) Size of mass and presence of effusion d) Whether lymph nodes are UL or contralateral e) Distance from carina 3. Progress of COP 2 years after initial Dx on lung wedge resection a) Resolution b) worse with increase subpleural honeycombing c) variable - 30% worse, 50% better, 40% stable d) resolution of alveolitis but persistent honeycombing 4. Prominent pulmonary arteries, least likely cause a) smoker with basal emphysema b) patient with erosive arthropathy c) DM with CRF d) 2y.o. with PDA e) SLE 5. Asbestosis occupational lung exposure with complications: Which is atypical? a) non ca++ plaques b) pleural effusions c) fibrosis d) mediastinal nodes e) diffuse pleural thickening mediastinal nodes are a feature of mesothelioma (StatDx) but not asbestos plaques or asbestosis. A later recall has a more atypical option to choose. 6. Non specific pulm fibrosis on bx: atypical cause a) Chlamydia pneumonia Page 2

b) Sarcoid c) Mason sanding occupation d) Rheumatoid 7. 2 yearly HRCT. Not typical cause of pulm changes a) SLE b) RA c) PAN d) Wegeners 8. R hilar mass with histology showing high cellularity, intercellular bridges, eosinophilic cytoplasm. Cause: a) SCLC b) SCC c) AdenoCa d) Bronchogenic cyst 9. Carcinoid which is atypical a) bronchiectasis b) 8cm cystic lesion c) lobar collapse d) endobronchial location 10. Fat embolism features are not a) CNS, skin changes b) Headache c) Onset within 6/24 d) Cerebral haemorrhage 11. Legionella is a) Gram neg bacillus b) Gram pos cocci c) Fungus d) Parasite 12. Cryptococcus neoformans atypical changes include a) solitary 3cm lung mass b) milary disease or multiple nodules c) basal ganglia lesions d) basal meningitis e) Vasculitis in basal cerebral vessels -StatDx: no mention of vasculitis for brain crypto 13. Which are not true: a) ABPA due to colonisation of aspergillus in bronchi b) Halo lesions invasive aspergillosis due to central cavitation 14. Re: PE, which is true A) pulmonary haemorrhage due to reperfusion of infarcted lung B) 30% recur after 1st DVT C) equal distribution in all lobes

Page 3

15. Symptoms of asthma and granuloma on Bx. Cause: a) sarcoid b) hypersensitivity c) BOOP d) Bronchiolitis obliterans 16. RE TB a) Mantoux test cant tell prior infection from active infection b) 1/10000 of primary infections are symptomatic c) Mantoux test is negative in overwhelming infection 17. Centrilobular emphysema affects the: a) All airway distal to terminal bronchiole b) Alveolar sacs only c) alveolar ducts but not alveolar sacs 18. RE Mesothelioma a) 10-20% have asbestosis b) plaques have asbestos bodies neither of these statements are correct April 2005 Path 19. Concerning fat embolism, which of the following statements is least correct? a. Over 80% of fat emboli are asymptomatic b. Features of fat embolism include hypoxia, thrombocytopenia, and CNS symptoms c. symptoms are usually established within 6 hours of trauma/fracture d. neurological manifestations include irritability, restlessness, confusion, and coma e. 20-50% of cases are associated with purpuric rash 20. The pathogenic agent of Legionnaires disease is L. pneumophilia. This agent is best described as a? a. gram negative bacteria b. spore forming coccus c. helminth d. protozoa e. richettsia 21. 35 yo outpatient HRCT request Late onset asthma, worsening dyspnoea, initially episodic, and now chronic. Biopsy shows granuloma:. Which of the following is most likely with this history? a. Silicosis b. TB c. Hypersensitive pneumonitis d. Sarcoid e. Histoplasmosis 22. Concerning centrilobular emphysema, which of the following is most correct? a. The acini are uniformly enlarged from level of terminal bronchiole to terminal alveoli b. proximal part of acini enlarged, relative or complete sparing of distal acini c. proximal portion of acinus normal or near normal, with dominant involvement of distal portion d. whole acini destroyed leaving irregular lined spaces greater than 1 cm in diameter e. acini uniformly involved but disease effects central zone of secondary pulmonary lobules Page 4

23. Pulmonary adenocarcinoma vs squamous cell carcinoma. Which of the following is most correct? a. The two are macroscopically similar b. adenocarcinomas are more likely to show cavitation c. adenocarcinoma is more likely to be peripheral, and affect women d. adenocarcinomas have a stronger associated with emphysema/chronic airway disease e. adenocarcinoma is more likely to show massive lymphadenopathy and extrathoracic disease compared with SCC 24. Patient with a markedly thickened pleura encasing one lung. Which of the following statement is most correct? a. Only approximately 10-20% of people with mesothelioma will have associated lower lobe fibrosis b. mesothelioma and metastatic adenocarcinoma have markedly different appearances on light microscopy c. presence of asbestos bodies on pleural biopsy suggest change is more likely reactive/fibrosis d. granuloma in pleural tissue suggests chronic irritation rather than a neoplastic +/- infective aetiology e. apical lung lesion suggests it is most likely infective in nature 25. A chest CT request states: pulmonary carcinoid. Which of the following indings would be least expected? a. Bronchiectasis b. 1cm endobronchial mass confined to a mainstem bronchus c. 8 cm cystic mass d. Pneumonia e. hilar lymphadenopathy (25%, more common in atypical) 26. Concerning PE, which of the following is most correct? a. Approximately 40% lead to infarction b. emboli affect upper, mid and lower zones equally but not lobes equally c. in the young, PE more frequently leads to infarction d. In the presence of a predisposing cause, secondary episodes may occur up to 30% e. pulmonary haemorrhage in PE implies reperfusion of an infarct 27. 57 yo. Man has a non specific pulmonary fibrosis. Which of the following is least likely to be related? a. past employment as stone mason b. history of an erosive arthropathy c. history of chlamydial infection d. history of previous chemotherapy e. muscle biopsy - non-caseating granuloma 28. Least likely finding in invasive aspergillosis a. Cerebral vessel changes b. Can mimic mucormucosis c. Lung lesion can resemble a target due to central cavitation d. involvement of brain, kidneys, heart valves 29. A patient has prominent pulmonary artery, which of the following Page 5

histories is least likely cause of pulmonary arterial hypertension? a. 38 yo smoker with basal emphysema b. 32 yo female whose mother died of primary pulmonary arterial hypertension c. 2 yo refugee with pansystolic machinery type cardiac murmur d. 30 yo female diabetic with chronic renal failure e. 22 yo with SLE with recurrent right leg swelling 30. 62 yo smokers CXR is introduced at a clinical meeting as showing occupational asbestos exposure with complications. Which of the following is least likely to be an associated finding? a. focal non-calcified pleural thickening b. pleural effusion c. mediastinal lymphadenopathy d. lobar collapse e. progressive massive fibrosis 31. Least likely to have assoc pulmonary findings on HRCT a. SLE b. Progressive systemic sclerosis c. RA d. PAN e. Dermatomyositis polymyositis 32. CXR report states, right hilar mass invading mediastinum, associated with hilar, mediastinal lymphadenopathy but no subcarinal disease. No other metastasis is seen. What else would be necessary for ISSLC TNM staging? a. size of lung lesion b. sides of hilar and mediastinal nodes involved c. size of mass and presence/absence of pleural effusion d. size of mass and presence of distal lung collapse e. distance of the lung hilar mass from the carina 33. Primary TB a. only 1/10,000 primary infection is clinically significant b. 80% patients asymptomatic c. progressive primary disease most commonly causes middle or lower lobe consolidation 34. The main abnormality in Cystic fibrosis is : a. Transmembrane electrolyte transport abnormality 35. Pulmonary embo]ism most correct statement a. haemorrhage suggest infarction see later question, may be a better answer 36. Multiple lung lesions, the working diag is wegener's granulomatosis, the following is most against the disease a. C-ANCA negative b. lack of renal involvement 37. Tim a. is a legend b. is a great bloke Page 6

c. has enormous genitalia d. all of the above Path MCQ SYDNEY 38. Which is least likely to be associated with sarcoid? A. African American B. Asian C. Ashkenazic Jewish European D. Male 39. What is least likely finding on CXR if pt has restrictive cardiomyopathy ? A. Bronchiectasis B. Bilat hilar lyphadenopathy C. Basal bullous disease D. ? fibrosis ?scleroderma 40. ARDS lung changes persist for a long time because ? A. Endothelial damage B. Organisation and fibrosis often end result not resolution. 41. definition of chronic bronchitis A. productive cough for >3/12 for 2 consecutive years 42. Posterior mediastinal tumour, most likely ? A. Gangliocytoma B. ganglioneuroma 43. Pleural plaques, least correct ? A. histologically see asbestos bodies 44A. 40 yo woman with enlarged pulmonary arteries, and pathologically a web like network of capillary formations within the vessel wall and lumen, is most likely secondary to ? A. primary pulmonary hypertension B. recurrent pulmonary emboli 44B. Types and percentage of lung Ca A.SCC 25-40 %, Adeno 25-40%, Small Cell 20-25%, Large Cell 10-15% B.-E.other combinations Pathology MCQ April 2003 45. Patient with suspected psittacosis for HRCT A. Inflammation- histiocytes /lymphocytes confined to alveolar walls B. Neutrophilic exudate within alveoli C. Grey / white slough on bronchial walls D. Mucous plugging and areas of atelectasis 46. Patient with HIV with normal CD4 count, patchy pulmonary opacities on CXR, which is most likely? A. CMV Page 7

B. Toxoplasmosis C. Kaposis D. Cryptococcus E. PCP None of the options are great CD4>500-bact, tb, ca; 200-500-recurrent bact, tb, lymphoma; <200-pcp, ks, cmv, mai, dissem fungal 47. In pulmonary emboli, which is true A. Haemorrhage implies infarction B. Always infarct as vessels are end arteries C. With haemorrhage underlying parenchyma is preserved Robbins page 742 48. In pure emphysema versus pure chronic bronchitis A. Emphysema occurs younger B. Chronic bronchitis more likely to have cardiomegaly C. Elastic recoil is preserved in emphysema 49. Coal workers pneumoconiosis, which is not true D. 2-5 x risk of lung cancer independent of smoking E. <10% go on to Progressive massive fibrosis F. Characterised by coal macules and coal nodules G. No increased risk TB 50. In benign asbestos pleural plaques, which would be most atypical A. Pleural effusion B. Predominant anterior and posterior plaques C. Diaphragmatic involvement D. Parietal pleura E. Golden brown nodes with iron containing proteinaceous material 51. Which does not cause Bronchiolitis Obliterans A. Tuberous sclerosis B. GVHD C. Collagen vascular disorders D. Viral infections E. Inhaled toxins 52. In Goodpastures syndrome A. Lung involvement precedes renal involvement B. Lung involvement occurs later than renal C. Lung involvement characterised by necrotizing haemorrhagic pneumonitis D. Usually mild Robbins page 541 53. Small cell carcinoma, which is best A. Also known as oat cell B. Neuroendocrine granules and most likely to have Para neoplastic syndrome C. Treatment is with lobectomy and chemotherapy D. Usually peripheral but has hilar nodes Page 8

E. Not associated with smoking R762 - dense-core neurosecretory granules 100 nm in diameter in two thirds of cases 54. Definition of Chronic Bronchitis A. Clinical + Micro/Macro B. Reid Index >0.4 C. Histo Definition D. Histo which was Bronchiectasis *Defined clinically 55. Mass in lung, with pericardial involvement A. T4 B. T3 C. N2 D. M1 E. Mesothelioma 56. Man doesnt want surgery, hilar mass, oncologist wants to know if small cell, does CT most likely bit of history to help A. CXR 1 yr ago, hilar mass, lost to follow up B. Normal CXR 9/12 C. ACTH and ADH, PTH Robbins page 763 ACTH/ADH SCLC, PTH - SCCa 57. CF A. Late onset recognized as chronic pancreatitis B. If significant liver disease, consider alternative Dx C. Mg co-factor for Cl channel D. another liver stem. 58. Mesothelioma true a. Invol parietal and visceral b. Smoking c. Lag time variable 5-15yrs d. EM to Dx from adenoCa e. Heavy exposure StatDx- primarily involves parietal pleura 59. Silicosis least likely. Hx of retired miner a. Upper fibrosis b. PAP c. Increased risk TB d. Cancer (no proven assoc ca) e. Infection? 60. Hodgkin Disease, 50yo, with Chemo, develops lung parenchymal mass. a. CMV b. HD recurrence c. Primary TB d. Secondary TB extranodal HD uncommon, 50yo immunsupressed with chemo might get postprimary TB

Page 9

61. Community acquired pneumonia least correct a. Viral b. Mycoplasma & Chlamydia c. Px with symptoms of dyspnoea, hemoptysis, effusion d. Bronchial ulceration means bacterial superimposed infection? Robbins page 751 62. Carcinoid a. All benign b. Rarely deviates the bronchus or invades it. c. Involves type 1 pneumocytes d. Difficult to make Dx on FNA e. Small Cell Ca. 63. Thought pt has Wegners, but Chest normal most likely evidence to support Dx. a. cANCA b. Nasal ulceration c. Anti mitochondrial antibodies 64. Sarcoid polyclonal.. on Bx, most correct a. Need to revist pathology b. Crohns c. MALT d. Never involves extraabdominal LN Sarcoid is polyclonal. Crohns is also. Best answer from the ones available. 65. TB most true? a. organ involved can be CNS or kidney in isolation b. type 1 hypersensitivity reaction c. Ghon focus is parenchymal + LN d. Biopsy caseous region vs non necrotic region for Dx of Bacilli for Rx Pathology MCQ September 2002 66. Pulmonary fibroma. Unusual finding A. Slightly mobile and on pedicle B. 1-2 cm in size C. 8-10 cm in size D. associated with pulmonary effusion E. cystic formation with the mass 67. Wegeners granuloma, commonest site A. Lung B. Paranasal sinus C. Nasopharynx D. Kidneys E. Skin 68. 52-year-old female, non-smoker, 3cm pulmonary mass- query cause? A. Primary adenocarcinoma B. Primary SCC C. Primary small cell Ca Page 10

D. Primary bronco alveolar E. Metastatic from breast 69. Bronchial carcinoid A. 0.1-0.05 % lung tumours B. 5th 6th decade of life C. 2-3:1 male: female D. 10% atypical, 50% carcinoid recurrent E. Histology keratin pearls (1-5% of all tumours) (most <40) (M:F=equal) 70. Coal workers pneumoconiosis on 6 months follow up chest x-ray least likely finding A. TB B. PMF C. Bronchial Ca D. Kaplans syndrome E. Pulmonary HT 71. Sarcoid, what is true answer? A. Subpleural fibrosis and honeycombing B. Hypo gamma globulinemia C. Subpleural interstitial fibrosis D. Bone lesion distal and proximal femur E. 50% affect spleen and liver 72. Desquamative interstitial pneumonitis A. Hamman rich syndrome B. Desquamative lymphocytic interstitial pneumonitis C. Chronic interstitial pneumonitis D. Treatable condition and can progress to UIP E. Chronic form of UI 73. Young girl with moderate to severe sarcoid presents with sudden death cause? A. Renal failure B. Pulmonary HT C. Ruptured berry aneurysm D. Hypertensive cerebral haemorrhage E. Cardiac sarcoid 74. Definition of Bronchiectasis A. Permanent dilatation bronchus and bronchioles > accompanying arterioles B. Permanent dilatation bronchus and bronchioles > 1.1 accompanying arterioles C. Necrotizing infection of bronchus and bronchioles D. Chronic infection of bronchus and bronchioles E. Cylindrical saccular, varicose dilation of bronchus and bronchioles 75. What is the most common cause of brain abscess in adult A. Streptococcus & Staph B. Staph & TB C. Staph & Toxoplasma D. TB & Nocardia E. PML Page 11

76. Chronic alcoholic presents with nystagmus, ophthalmoplegia, ataxia, but with preserved speech, no facial droop. What is likely diagnosis? A. Wernickes encephalopathy B. Marchiafava bignami syndrome C. Korsakoffs syndrome D. Cerebellar infarct E. Osmotic pontine myelinosis 77. Commonest cause of community-acquired pneumonia a. Klebsiella b. Staph aureus c. Strep pneumonia 78. Lung carcinoma within 2cm of carina, involvement of pleura, pericardium, diaphragm or collapse of lobe. a. T3 b. T4 c. Stage II d. Stage III e. Not enough information. 79. Most diagnostic of asbestosis a. Asbestos bodies b. Interstitial fibrosis 80. Path a. blows b. rocks 81. Ca lung most correct a. Small cell better prognosis than squamous b. Squamous commonest c. Large cell = scar carcinoma 82. Which of the following does not show neuroendocrine differentiation a. Small cell tumour b. Typical Carcinoid c. Tumorlets d. bronchioloalveolar cell carcinoma e. atypical carcinoid 83. Lobar pneumonia most common important pathogen a. Strep pneumonia b. Klebsiella c. Staph d. PCP 84. Most characteristic of TB a. Caseous necrosis b. Acid fast bacilli c. fibrinoid necrosis Page 12

85. Lung mets commonest cause a. Colorectal b. Renal c. Breast d. Gastric e. Head and neck f. Thyroid 86. Immune reaction to tuberculosis is an example of a. Antibody mediated immunity b. Delayed hypersensitivity reaction c. Immune complex mediated immunosuppression 87. Which is not a feature of Alzheimers a. Hirano bodies b. Lewy bodies c. Senile Plaques d. Neurofibrillary tangles e. Granulovacuolar degeneration f. Amyloid Lewy=Parkisons 88. Secondary TB can occur in all of these settings except a. reactivation b. reinfection c. Ghon complex d. Immune suppression e. Silicosis 89. Characteristic histology features of asbestos fibres Asbestos bodies appear as golden brown, fusiform or beaded rods with a translucent center and consist of asbestos fibers coated with an iron-containing proteinaceous material. They arise when macrophages attempt to phagocytose asbestos fibers; the iron is presumably derived from phagocyte ferritin. 90. Differential diagnosis of malignant mesothelioma includes all of the following except: a. solitary fibrous tumour b. metastatic adenocarcinoma c. pleural fibrosis d. pleural plaques e. haemangioblastoma 91. Emphysema is defined as: a. destruction of airspace distal to terminal bronchioles with destruction of walls b. destruction of airspaces proximal to distal bronchioles with no destruction of walls c. dilated bronchioles d. subpleural fibrosis 92. Small cell tumour a. typical carcinoid

Page 13

93. Which of the following is least likely associated with mesothelioma a. Exposure to chrysotile fibres b. exposure to amphibole fibres c. latency period between exposure and tumour d. pulmonary fibrosis Crocidolite (=amphibole) >>> Amostie (=another amphibole) >>> chrysolite (serpentine). 94. Regarding pulmonary emboli and infarcts, which of the following is MOST CORRECT: a. Majority result in white infarcts because it is an end circulation b. 75% are in the lower lobes c. Pulmonary haemorrhage suggest infarction d. The presence of viable alveoli pneumocytes/endothelium in an area of haemorrhage is against a diagnosis of PE e. >50% of infarcts are solitary PATHOLOGY MCQS APRIL 2003- BRISBANE 95. CHRONIC BRONCHITIS VS EMPHYSEMA MOST CORRECT a) cardiomegaly > in chronic bronchitis b) bronchitis earlier and more short of breath c) cor pulmonal more common in emphysema d) CB chronic hypoxemia and hypercapnia, pulmonary HTN and cor pulmonale 96. ASBESTOS PLAQUES LEAST ASSOCIATED WITH a) asbestos bodies b) pleural effusions c) anterior/posterolateral site d) diaphragm 97. GOODPASTURES lung changes a) may avert pulmonary disease with renal treatment b) minimal, same type as ? c) severe pulmonary disease secondary to vasculitis d) pulmonary disease secondary to granulomatosis 98. Least likely in Coal Workers Pneumoconiosis a. coal macule b. coal nodule c. PMF d. Increased incidence of TB e. 2-3 X increased incidence of Ca. Path MCQ 2002 COUCHER 99. Desquamative Interstitial Pneumonitis is a) An early form of UIP with a better prognosis no correct answers in this recalled question 100. The following are not associated with primary TB a) Consolidation b) Lung disease of secondary TB worse than primary Page 14

c) Ghon complex defined as the pulmonary lesion only 101. Pulmonary carcinoid a) Commoner M:F 2:1-3:1 no correct answers in this recalled question 102. Pathological definition of bronchiectasis a) Airways as large as arterioles b) Airways 1.1x arterioles c) Dilatation of bronchi and bronchioles d) Necrotising persistant. 103. Exams a) are awesome. Why cant we have them every year? b) suck the life out of you and leave you bitter and old. 104. Features NOT compatible with pleural fibroma are a) size 1-2cm b) size 8-10cm c) pleural effusion d) narrow pedicle e) cystic components PathologySept2004 105. primary bronchogenic carcinoma. Which of the following has no or least relevance to staging? a) Tumour size of 4 or 6 cm b) Involvement of regional intrapulmonary nodes c) Side of any mediastinal node enlargement d) Distance from the carina e) Presence of lobar collapse 106. TB, which is false a) Primary TB characterized by the ghon focus b) TB bronchopneumonia characterized by 3-5 mm yellow nodules c) Cavitation is characteristically seen in reactivation of tuberculosis 107. Asbestos fibres most likely to be found in : a) Malignant mesothelioma b) Pleural fluid c) Pleural plaque d) Parenchymal biopsy e) Bronchial washings 108. wegeners, which is against the diagnosis a) involvement of nasal mucosa b) renal involvement c) cANCA d) female e) necrotizing vasculitis f) necrotizing glomerular nephritis g) neg ANCA Page 15

h) granulomatosis of upper and lower resp tracts 109. 67 year old man, 4cm small cell lung Ca, hilar lymph node, no mets - query survival (without treatment)? a) 20% 5 years b) 50% 1 year c) 50% 6 months for untreated limited disease d) 6-18 months PATH MCQ COMBO - 2002 110. Regarding Sarcoidosis a) Liver and spleen involvement in more than 50% b) Has lesions in the distal ends of the long bones Histopathologically not radiologically 111. Hyaline Membrane Disease (mild form) a) Child typically breathless at birth b) Child becomes breathless a short time after birth c) Various durations for recovery 112. Pulmonary carcinoid a) Commoner M:F 2:1-3:1 no correct answers in this recalled question 113. Paraneoplastic syndromes do NOT include a) Unilateral arm weakness from peripheral lung Ca b) Myoclonus with Ca breast Robbins counts pancoast tumors in with paraneoplastic syndromes. Lambert-Eaton myasthenic syndrome in which muscle weakness is caused by auto-antibodies (possibly elicited by tumor ionic channels) directed to the neuronal calcium channel. Breast CA ! hypercalcemia. Hypocalcemia causes myoclonus. 114. 8yr old girl having anaesthetic for MRI to investigate choreiform movements. Murmur heard by anaesthetist most likely due to a) Previous rheumatic fever Sydenham chorea 115. Features NOT compatible with pleural fibroma are a) size 1-2cm b) size 8-10cm c) pleural effusion d) narrow pedicle e) cystic components 116. Causes of aortic dissection do NOT include a) Hypertension b) Marfans syn c) Atherosclerosis d) Pregnancy Page 16

e) Cystic medial degeneration 117. 4cm right hilar small cell carcinoma detected. The prognosis without treatment is a) 50% alive at 6 months b) 50% alive at one year c) 20% alive at 5 years XPATH MCQ COMBO 2003 APRIL 2003 UPDATE 118. HIV pt with normal CD4 count presents with patchy ill defined infiltrate on CXR. Most likely? A. PCP. B. Toxo C. CMV D. Kaposi E. Cryptococcus Most common at normal CD4 would be bacterial pneumonia, but not an option here 119. Most likely characteristic of Goodpastures ? A. Causes a necrotising interstitial pneumonitis B. Renal changes predate lung changes C. Lung changes predate renal changes 120. Thymoma a. contains cells of epithelial origin only 121. ARDS lung changes persist for a long time because ? A. Endothelial damage B. Organisation and fibrosis often end result not resolution. 122. Posterior mediastinal tumour, most likely ? A. Gangliocytoma B. ganglioneuroma PATH APRIL 2004 123. Granulomas incorrect a. HIV b. Sarcoid c. TB d. Berylliosis 124. All of the following are pneumoconioses except a. Silicosis b. Asbestosis c. heavy metal lung d. Berylliosis e. Bird fanciers lung Robbins blurs the lines on this a bit p732 [The term "pneumoconiosis" was originally coined to describe the non-neoplastic lung reaction to inhalation of mineral dusts encountered in the

Page 17

workplace. Now it also includes diseases induced by organic as well as inorganic particulates and chemical fumes and vapors.] 125. a1-ATdefic incorrect a. A Recessive b. Von Meyenberg complexes c. End stage get cirrhosis + cholestasis Von Meyenberg = bile duct hamartoma 126. Wegeners granulomatosis is characterised by all of the following except a. generalised vasculitis b. GN c. Granulomatosis of the upper and lower respiratory tract d. ANA +ve e. ANCA +ve 2005 APRIL PATHOLOGY RECALLED MCQS 127. Pathologist is looking for a gross specimen of adenocarcinoma of the lung to show medical students. He has a specimen of SCC only. RE: Adenocarinoma vs SCC lung c) similar macroscopically d) adeno more common in female, less associated with smoking 128. Non specific pulm fibrosis on bx: atypical cause e) Chlamydia pneumonia f) Sarcoid g) Mason sanding occupation h) Rheumatoid 129. 2 yearly HRCT. Not typical cause of pulm changes e) SLE f) RA g) PAN h) Wegeners 130. R hilar mass with histology showing high cellularity, intercellular bridges, eosinophilic cytoplasm. Cause: e) SCLC f) SCC g) AdenoCa h) Bronchogenic cyst PATHOLOGY MULTIPLE CHOICE QUESTIONS APRIL 2005 CANBERRA 131. Regarding Mycobacterium tuberculosis, which of the following is least correct? a. Risk factors include HIV, chronic renal failure b. Mantoux test does not differentiate between active disease and previous exposure c. Mantoux test may be negative in severe active disease d. 1: 10 000 develop active disease e. in immunosuppressed individuals, disease looks like postprimary disease, involving middle to lower lobes Page 18

132. A clinician wants a CT scan demonstrating adenocarcinoma for teaching medical students. He already has an example of squamous cell carcinoma. Which of the following is most correct? a. Adenocarcinoma is most frequently peripheral, in women, non-smoking individuals b. Adenocarcinoma is often central c. Adenocarcinoma is associated with smoking d. Adenocarcinoma demonstrates cavitation 133. Regarding Legionelia pneumophilia, which is the most correct? a. Gram negative bacilli b. Spore forming coccus c. Heiminth d. Protozoa 134. A CT scan report reads: "Right hilar mass with involvement of the mediastinum, and mediastinal and hilar lymphadenopathy, with no subcarinal lymphadenopathy. No distant metastases." What other information is required for accurate staging according to the ICLS? a. Size of the lung mass b. Side of the mediastinal and hilar lymphadenopathy c. Size of lung mass and pleural effusion d. Size of lung mass and lobar collapse e. Distance of lung mass from main stem bronchus 135. Patient presents for follow up HRCT. 2 years ago a lung biopsy demonstrates cryptogenic organising pneumonia. What is the likely result? a. Progressive subpleural fibrosis at both lung bases b. Disease progression in upper and mid zone distribution c. Variable - 40% worse, 30% stable, 30% improve d. Improvement or complete resolution e. Progressive subpleural honeycomb in the upper zones 136. Clinical history of "Late onset asthma. Previously episodic, now continuous dyspnoea. Biopsy shows granuloma." Given the clinical history, which is the most likely diagnosis? a. Silicosis b. Tuberculosis c. Hypersensitivity pneumonitis d. Sarcoidosis 137A. CXR with enlarged pulmonary artery. Which of the following is least associated with pulmonary arterial hypertension?" 137B Regarding pulmonary embolism, which is the most correct? a. 40% of pulmonary embolus results in pulmonary infarction b. Pulmonary infarcts develop in young individuals because of poor collateral circulation c. 30% develop recurrent pulmonary embolus have a predisposing cause d. Pulmonary haemorrhage implies infarction 138. 45yro male. Pulmonary infiltrates, chronic middle ear infections and renal changes. What is most likely? a. Wegeners granulomatosis b. PAN

Page 19

139. Diffuse pleural thickening in patient with heavy asbestos exposure. A mesothelioma less likely if apical cavitating lesion suggests infective aetiology b. more likely to be reactive if biopsy does not show any asbestos bodies 140. Which of the following regarding ABPA is false? a. when invasive ABPA can mimic mucormycosis b. when invasive ABPA can have central haemorrhage c. colonization occurs in bronchi assuming they meant invasive aspergillus rather than invasive ABPA 141. Calcification of pulmonary arteries on imaging - least likely to occur in a. SLE b. 2 year old with pansystolic murmur C. RA (too young, longstanding) 142. Pt with carcinoid of the chest, least likely finding? a. endobronchial mass b. Large cavitating parenchymas mass (80%central, cavitation is rare, 1-4cm) XTHE COLLIMATION FINAL ANSWERS 2 143. Emphysema a) centriacinar emphysema constitutes 95% of cases No numbers in Robbins that I can see. Sounds correct. 144. Silicosis / CWP a) Lung cancer b) Lower vs upper c) Lymph node calcification Went for this one assuming the question was whats the difference between them. 145. Definition of Bulla a) >1cm and <1mm thick wall 146. Which has the highest incidence of pulmonary fibrosis. a) Scleroderma b) Masturbation > 2x/day

CNS
April 2005 Path MCQ 1. Reason for MS plaque distribution perpendicular to the lateral ventricles a. reflects major fibre tracts b. periventricular veins c. periventricular plaques limited by the optic radiations d. periventricular lesions predispose to extension leading to chaining 2. Patient with possible ADEM. Which clinical setting least likely? Page 20

a. 2 week history of viral illness b. female c. headache and confusion progressing to coma in 48 hrs d. HIV positive e. Age less than 40 Tough one that nobody agrees on. Requisites (p347) mentions HIV, Robbins, Osborn, StatDx doent. Only one case report in the literature (Neurology, 2001) and that discusses the unlikeliness of this scenario due to impaired cell mediated immunity. 3. A new Nuc Med agent can attach to the amyloid in neuritic plaques. Which is most correct? a. Should help exclude congophilic angiopathy in the elderly b. Uptake would allow distinction between Alzheimers and age matched Parkinsons disease c. Alzheimers patients should have greater uptake in the medial temporal lobe than age matched control patients d. Cerebellar uptake would suggest ataxic telangiectasia e. Deep cerebral uptake would suggest multi-infarct dementia or MS 4. An unconscious alcoholic with multiple medical problems is resuscitated with IV fluids and thiamine. MRI shows white matter lesions in pons, tegmentum and deep cerebral white matter. Which is most likely? a. Central pontine myelinolysis b. Beri-beri c. Wernicke korsakoffs d. ETOH encephalomyelitis e. Severe combined degeneration of the white matter 5. 34/40 fetus with large mass protruding posteriorly from sacrum. No evidence of Chiari malformation. Most likely is a. benign sacrococcygeal teratoma b. malignant sacrococcygeal teratoma c. congenital neuroblastoma d. imperforate cloacal membrane e. mature ganglioneuroma 6. 16 year old with Freidrichs ataxia has a poor quality MRI. Most likely cause is a. intention tremor b. recurrent facial tic c. orthopnoea d. salaam spasms e. hemiballismus Orthopnoea is due to the associated cardiomyopathy there is multifocal destruction of the myocardial fibres +/- pericardial adhesions (present in 50% of autopsies) 7. Hemorrhagic areas in cerebrum on CT. Least likely a. Recent pelvic fracture b. Past rheumatic fever c. Active mastoiditis d. Recent neck manipulation e. Recent placental abruption 8. 65 year old, 3rd yearly follow up scan for CJD. Which is most correct? a. This is expected as CJD is slowly progressive Page 21

b. Incorrect diagnosis c. Patient most likely has variant CJD d. CJD has variable progression with 10-15% having a long term survival of > 10 years e. Patient more likely to have the more indolent familial form 9. Least likely sites for Toxo in the brain a. GW junction b. Pons c. Cerebellum d. Spinal cord e. Putamen Robbins p1379 10. Most at risk for cerebral venous infarction a. diabetic in renal failure b. post obstetric patient c. young girl on progesterone only pill d. young girl with SLE 11. Solitary 3cm cystic lesion in the brain containing an opaque gel-like substance a. Hydatid / echinococcus b. Cysticercosis c. Amoebiasis d. Strongyloides e. Ascaris On size this is the most likely as far as I can work out. Still not quite right though. 12. White matter disease least likely a. Picks b. ADEM c. DAI 13. Manifestations of Cryptococcus in immunocompromised. Which is least likely? a. Granulomatous arteritis of Circle of Willis b. Cystic spaces in basal ganglia c. Miliary (5mm) lung nodules d. Basal meningeal enhancement e. 3cm pulmonary mass with no hemorrhage around it option C contradicts itself with the miliary and 5mm but could be recall error. November2003 14. Most correct statement about DNET is : a. degeneration of an anaplastic astrocytoma b. cystic change and haemorrhage Both wrong Path MCQ SYDNEY 15. CNS tumour least likely to be cystic? A. Meningioma Page 22

B. Acoustic Schwannoma C. Haemangioblastoma D. Pilocytic astrocytoma E. DNET 16. Most likely cerebral tumour with periventricular extension in an immunocompromised patient? A. GBM B. Lymphoma Pathology MCQ April 2003 17. In alcoholic with hyponatremia and partial paresis, which is most likely A. Central pontine myelinosis B. Pontine myelinosis, midbrain and supratentorial changes C. Pontine lesion secondary to Thiamine deficiency D. Subacute combined degeneration of cord secondary to thiamine deficiency E. Cord degeneration secondary to folate deficiency 18. Multiple sclerosis, least likely finding is A. Cerebellar white matter cerebellar vermis B. Corpus callosum C. Peri venular 19. Mycotic aneurysms A. Rarely bleed B. Peripheral arterial C. Circle of Willis 20. Patient with drug resistant Parkinsons and autonomic neuropathy A. Shy drager B. Drug resistant Parkinsons C. Striatonigral degeneration D. Olivopontocerebellar atrophy E. Progressive supranuclear palsy Referring to Parkinson plus syndromes AKA multisystem atrophy Poor response to dopamine replacement therapy More severe clinical manifestation of parkinsonism

Shy Drager- autonomic failure + cerebellar and extrapyramidal disturbances Progressive supranuclear palsy- axial rigidity, supranuclear gaze palsy, pseudobulbar (UMN CN 9, 10, 12- involves medulla) signs. Marked midbrain and tectal atrophy Olivopontocerebellar atophy- denegerative condition characterized by atrophy of pons, medulla, vermis, cerebellar hemispheres and middle peduncles. Striatonigral degeneration- Parkinson like syndrome with atrophy of putamen 21. Picks disease, uncommon findings A. Asymmetrical atrophy Page 23

B. Predominant frontal lobes C. Cortical atrophy D. Involvement of post superior temporal gyrus & parietal lobe 22. Periventricular mass in patient renal transplant A. Primary lymphoma B. GBM C. Secondary lymphoma 23. 40 year old female with stroke, underlying cause least likely is A. Atherosclerosis B. Dissection C. Coarctation of aorta D. Giant cell arteritis E. Mitral valve prolapse September 2005 PATHOLOGY MCQS 24. Central Pontine Myelinosis least likely a. Esophageal varices b. Cord involvement c. Resuscitation with IV fluids in hypotensive d. Supra cortical involvement e. Recent Quadrapresis 25. Meningioma least correct a. WHO 1 classification b. Intra atrial position c. Invasion into brain parenchyma does not change grade d. Papillary ?necrosis 26. 15 yo with psychosis, MRI showed diffuse WM abnormality a. Metachromic Leukodystrophy b. ADEM c. Huntingtons d. HIV 27. Huntingtons chorea what does it show a. Various locations including basal ganglia b. Caudate nuc +/- putamen c. Cerebellum d. Locus Ceruleus 28. Craniopharyngioma on histology 60yo, solid 5cm, no cyst, no Ca2+ a. Review pathology - not a craniopharyngioma b. Typical cranio c. Papillary variant 29. Peripheral MCA aneurysm most likely? a. SLE b. PHx of irradiation for fibrous dysplasia c. HT Page 24

d. Hx of Rheumatic Fever or tooth extraction 30. Which is not associated with brain cortical involvement a. DNET b. Oligo c. Pleomorphic Xanthoastrocytoma All cortically based tumors. Option D must have been a cracker. Pathology MCQ September 2002 31. CJD what would be unusual finding A. Cortical atrophy is rarely found B. Survival of 3-3" years 32. What is the most common cause of brain abscess in adult F. Streptococcus & Staph G. Staph & TB H. Staph & Toxoplasma I. TB & Nocardia J. PML 33. Chronic alcoholic presents with nystagmus, ophthalmoplegia, ataxia, but with preserved speech, no facial droop. What is likely diagnosis? F. Wernickes encephalopathy G. Marchiafava bignami syndrome H. Korsakoffs syndrome I. Cerebellar infarct J. Osmotic pontine myelinosis K. Ganglioglioma L. Schwannoma versus plexiform neurofibroma M. GBM histology 34. Necrosis is a characteristic feature of a. Pilocytic astrocytoma b. Glioblastoma multiforme c. Acoustic schwannoma d. Craniopharyngioma e. Ependymoma 35. Solid mass involving the cortex of a temporal lobe is demonstrated on MRI. The most likely diagnosis is: a. DNET f. Pilocytic astrocytoma g. Anaplastic astrocytoma h. Mesial temporal sclerosis i. Pleomorphic xantho-astrocytoma 36. Least likely site for hypertensive bleed in the brain is: j. hippocampus k. cerebellum l. basal ganglia m. thalamus Page 25

n. putamen o. brainstem 37. BBB incorrect p. H2O soluble medium cannot pass if normal BBB q. Fat soluble medium cannot pass if normal BBB r. Capillaries are continuous s. Continuous capillaries have no fenestrations t. Tight junctions are important Sage p697. Continuous capillary walls in brain; tight junctions, lack of pinocytosis, absence of fenestrations. Lipid soluble molecules get through; water soluble do not. 38. Cortically based lesion which is true u. Oligodendroglioma v. Low grade anaplastic astrocytoma w. Corpus callosum x. Dandy walker 39. GBM what is the most important in Dx y. Necrosis z. Angiogenesis aa. Cystic change bb. Mitosis cc. Vasogenic oedema 40. Cavernous Angioma What is not typical a. detectable at angiography b. Bleeding tendency c. No intervening brain d. Pseudo capsule + surrounding hemosiderin laden macrophages e. Associated venous angioma 41. Which of the following is not a congenital CNS infection dd. chickenpox ee. rubella ff. CMV gg. Toxoplasmosis hh. Herpes PATHOLOGY MCQS APRIL 2003- BRISBANE 42. CNS tumour and cyst least likely - pilocytic astrocytoma - haemoangioblastoma - dnet - meningioma - schwannoma 43. Young patient and cva (42yo). Least likely cause - adpckd - ?Takayasus - gca Page 26

- fmd - dissection 44. Drug resistant parkinsons with cavitation and putaminal atrophy and autonomic dysfunction - Shy drager - Huntingtons - Drug resistant parkinsons 45. Hsv and cns changes. Most correct - Oncocytic intracytoplasmic inclusions - Age: 50-60 yo - Can present subacute with seizure, lethargy and ataxia inclusions are intranuclear 46. Picks disease and CNS least likely - Asymmetric changes - Putamen changes - Involvement of posterior temporal/parietal lobe Path mcq 2003 mohammed the mo 47. Patient with drug resistant parkinsons disease with involvement of autonomic nervous system, caudate, putamen A. Striao-nigral degeneration B. Shy drayger C. Huntingtons D. Drug resistant parkinsons 48. MS least common site A. Cerebellar white matter B. Cerebellar vermis C. Corpus callosum 49. Patient with cystic brain tumour ith solid component--- least likely a. Haemangioblastoma b. Pilocytic astrocytoma c. Meningioma d. Schwannoma e. DNET Path MCQ 2002 COUCHER 50. Commonest sites for ependymoma are - most commonly Fourth ventricle in children and spine in adults - Lateral and third ventricle in infants.. - Periventricular areas (permutations of adult/infant/children) 51. Microscopic features of Glioblastoma Multiforme do NOT include - Necrosis and areas of different histology 52. Features of Gangliocytoma include Page 27

- Superficial location 53. CNS features NOT typical of TB are - basal meningeal enhancement - 4cm focal mass - irregularity of vessels PathologySept2004 54. Hyperprolactinaemia, cause? a) Stalk compression 55. Huntingtons, which is true? a) AR b) Affects the putamen and caudate c) Presents in the second decade 56. A 24 yr old woman with Stage II lymphocyte predominate Hodgkin's lymphoma has been neutropaenic for 2 weeks in intensive care. Since her pre-treatment CT scan 3 months earlier her liver and spleen have increased in size and show new lesions up to 2 cm in size. Despite multiple antibiotics-she continues to decline and CT shows a new left MCA infarct. This is most compatible with d) Systemic candidiasis e) Bacterial endocarditis with systemic emboli /infarction f) Drug resistant lymphocyte predominate Hodgkin's lymphoma g) Transformation to a more aggressive lymphoma h) Non-bacteria thrombotic endocarditis with systemic emboli / infarction 57. Cystic brain lesion, least likely mass? a) Meningioma b) DNET c) Pilocytic d) Haemangioblastoma e) PNET 58. Atypical Scenario a) Craniopharyngioma in a 42 year old b) Anaplastic thyroid cancer in a 29 year old c) Bowel cancer in a 32 year old 59. Schwannoma vs neurofibroma, which is false? a) Neither goes on to cancer b) Neurofibroma contains axons c) Both contains schwann cells d) Schwannoma more separable from the nerve 60. Ependymoma least correct a) Fourth ventricle in kids b) Spine in adults (esp the conus) c) Fourth ventricle in all but more common in kids Page 28

61. Mycotic aneurysms ? a) Rarely bleed b) Peripheral arterial c) Circle of Willis 62. Concerning CNS demyelination, which of the following statements is correct: a) ADEM typically follows a bacterial infection b) Central pontine myelinosis is due to rapid correction of hyperkalaemia c) Multiple sclerosis lesions do not involve the corpus callosum d) Depletion of oligodendrocytes is a feature of MS lesions e) Multiple sclerosis increases in frequency with HIV 63. In drug resistant Parkinsons and autonomic neuropathy an MRI would look for a) striatonigral degeneration b) Shy dragger c) olivopontocerebellar atrophy PATH MCQ COMBO - 2002 64. HIV positive patient with CD4 count 100 has their first seizure. Most likely due to -PML -Toxoplasmosis -Lymphoma -Kaposis sarcoma 65. Alcoholic having treatment for Wernickes has rapid change in heart size over one week. Change most likely due to -Dehydration -Resolution of pericardial effusion -Projectional change on CXR -Beri-beri Thiamine deficiency: 3 syndromes occurring in this sequence: Polyneuropathy dry beri beri High output cardiac failure wet beri beri Wernicke-Korsakoff syndrome W-K syndrome: Opthalmoplegia, ataxia, and derangmentof mental function, confusion K psychosis: retrograde amnesia and confabulation. Affected areas; mamillary bodies, periventricular regions of thalamus, 4th vent, and cerebellum. 66. CNS features NOT typical of TB are -basal meningeal enhancement -4cm focal mass -irregularity of vessels XPATH MCQ COMBO 2003 67. SECOND RETINOBLATOMA TUMOUR OTHER SITES TO IMAGE Page 29

- pineal - cavernous sinus - cranial vault/sutures 68. Nasal polyps 69. Laryngeal Ca 70. Cholesteatoma ; which is most correct ? A. Stratified squamous epithelium with keratinous material B. contains cholesterol crystals 71. Most likely cerebral tumour with periventricular extension in an immunocompromised patient ? A. GBM B. Lymphoma PATH APRIL 2004 72. solid mass involving the cortex of a temporal lobe is demonstrated on MRI. The most likely diagnosis is: a. DNET b. Pilocytic astrocytoma c. Anaplastic astrocytoma d. Mesial temporal sclerosis e. Pleomorphic xantho-astrocytoma 73. Cortically based lesion ? true f. Oligodendroglioma g. Low grade anaplastic astrocytoma 74. Corpus callosum least correct a. Dandy walker 75. GBM what is the most important in Dx a. Necrosis b. Angiogenesis c. Cystic change d. Mitosis e. Vasogenic oedema 76. Which is not a feature of Alzheimers a. Hirano bodies b. Lewy bodies c. Senile Plaques d. Neurofibrillary tangles e. Granulovacuolar degeneration Amyloid 77. PNET which is the most typical appearances a. Cortical b. Angiogenesis ? c. Cystic Page 30

d. Vasogenic oedema e. Astrocytoma 2005 APRIL PATHOLOGY RECALLED MCQS 78. Which do not involve white matter a) Picks b) DAI c) MS d) ADEM e) alexanders PATHOLOGY MULTIPLE CHOICE QUESTIONS APRIL 2005 CANBERRA 79. Patient with "CJ diseasd'on MRI request. Comes for third annual follow up study. Which is true? a. CJ patients only live for average of 7 months b. Most likely has variant CJ disease XTHE COLLIMATION FINAL ANSWERS 2 80. CMV encephalitis a) Characteristic inclusions b) Ependymal and subependymal spread c) May cause haemorrhage 81. CJD and variant CJD a) caused by a slow virus b) CJD patients live for a few months whilst vCJD patients live for a few years c) Associated with frontal atrophy 82. Pilocytic astrocytomas a) Associated with NF2 b) 50% are solid c) Prognosis is less than 70 % 5 year survival 83. DNET a) intracortical b) degeneration of an anaplastic astrocytoma c) cystic change and haemorrhage 84. Least common site for meningioma a) adjacent to hippocampus b) parietal lobes c) between cerebrum and cerebellum. d) adjacent to nose 85. Berry aneurysm, most correct statement: a) majority occur at branch points b) 40% posterior circulation Page 31

86. Least likely site for hypertensive bleed in the brain is: a) hippocampus b) cerebellum c) basal ganglia d) thalamus

ENDO
1. Which is unlikely to be elevated in MEN I syndrome? a. Alk Phos b. Gastrin c. Ca d. Prolactin e. VMA 2. Hypertensive patient most likely findings a. 2cm adrenal Ca b. 10cm adenoma c. bilateral benign tumours d. bilateral hyperplasia e. adrenal atrophy secondary to sustained HTN 3. Persistent elevated PTH post surgery for hyperparathyroidism. Most likely cause a. Neoplasm producing PTH b. parathyroid adenoma in chest c. pituitary adenoma d. multiple parathyroid adenomas e. parathyroid hyperplasia 4. Regarding pheos a. T2 hyperintense b. Extra-adrenal pheos are less malignant than those associated with adrenal c. 2cm at Dx 5. Hyperprolactinaemia, cause? a. Stalk compression (controversial) hypothesis is that a large adenoma compresses the stalk and blocks the flow of inhibitory dopamine down the hypophyseal-portal system. 6. Causes of Addisons disease in order of importance? a) Autoimmune adrenalitis>Tuberculosis & Fungal>Metastatic malignancy>Diabetes b) Various orders of autoimmune adrenalitis, TB and fungal, Diabetes, Metastatic malignancy. 7. Increased calcitonin a) MEN I b) MEN IIA c) Medullary Ca 8. Incidence of various types of Thyroid cancer commonest to rarest are - Papillary, Follicular, Meduallary, Anaplastic and Lymphoma Page 32

Papillary carcinoma (75 to 85% of cases) Follicular carcinoma (10 to 20% of cases) Medullary carcinoma (5% of cases) Anaplastic carcinoma (<5% of cases) 9. Which of the following is least appealing? a. Failing report writing. b. Failing Path MCQs. c. Failing Radiodiagnosis MCQs. d. One night of passionate lovemaking with Wilma from caf Kaz / Zuki. 10. Causes of hyperthyroidism in order of importance a. Various orders of Graves, Toxic MNG, Toxic Goitre, Autoimmune thyroiditis 11. Male with elevated HIAA in urine and hilar mass has aortic and mitral murmurs most likely caused by -Right-sided cardiac valvular disease -Left-sided cardiac valvular disease -Hyperdynamic circulation -Paraneoplastic cardiac involvement - Right-sided heart failure resulting from right-sided endocardial fibroelastosis leading to tricuspid regurgitation and pulmonary valve stenosis Carcinoid heart disease; Robbins p599 12. Hypertensive patient with hypokalaemia has CT for screening. Most likely looking for -Adrenal mass <2cm -Adrenal mass >5cm -Bilateral adrenal enlargement 13. Least likely cause for hypothyroidism a. thyroidectomy b. congenital absence c. idiopathic d. radiation 14. Pt referred for imaging with Hashimoto's thyroiditis, most likely findings include a. pleomorphic ademoma with de differentiated to undiff CA or adenoCA b. haemorrhage in pleomorphic adenoma c. erosion and fistula .... 15. Hypoparathyroidism a) Associated with adrenal Path - Endocrine 16. Pt with cerebral oedema and neurological dysfunction, and lab results of dilutional hyponatraemia and high urinary sodium, the most likely cause is a. Small cell carcinoma of the lung b. EG of pituitary stalk c. Pituitary adenoma Page 33

d. Head injury with injury to the hypothalamus e. Sarcoidosis 17. Craniopharyngiomas the least accurate: a. arising from neuro-epithelial rests along the involuted hypophyseal-Rathke's duct b. most common non-glial tumor of childhood c. >50% occurs in childhood or young adults with a second peak in adults 40-60 years d. most often a cystic mass with a mural nodule e. Nodular or rim calcification in >90% of adult cases 18. 35 yo female with thyroid carcinoma & PHx of treated pharyngeal rhabdomyosarcoma. The least likely finding regarding the thyroid lesion is a. Orphan Annie nuclei b. Cytoplasmic invaginations c. Psammoma bodies d. broad sheets of amyloid e. Predominately Lymphatic mode of spread 19. Regarding Medullary Thyroid Carcinoma (MTC) - incorrect a. A malignant neuroendocrine tumor of parafollicular cell origin b. 25% are in association with MEN Syndromes c. cells are polygonal or spindle and disposed in nests separated by fibrovascular strorna. d. Most secrete calcitonin e. Familial tumors (non-MEN) are aggressive with a 50 % 10 year survival 20. The least correct statement is a. Thymic hyperplasia uncommonly associated with myasthenia gravis. b. Type 2 malignant thymomas often metastatic at presentation c. Critical feature in differentiating benign from malignant thymoma is the penetration of the capsule and invasion of adjacent structures d. Thymomas and those associated with myasthenia gravis are less aggressive. e. Parathymic syndromes are present in 40% of patients. 21. The least correct regarding Thymoma is a. The most common anterior mediastinal tumor in adults b. 30% are invasive (malignant thymoma) c. 35% of thymoma patients have myasthenia gravis d. 35% of myasthenia gravis patients have thymoma e. In patients with Aplastic anemia, 50% have thymoma f. Hypogammaglobulinemia - 15% have thymoma 22. Regarding Phaechromocytomas in MEN 2 (Sipple's Syndrome) - incorrect a. Bilateral in 50% b. metastatic behaviour is common c. often extraadrenal d. present in approximately 50% of patients e. post adrenalectomy 50% will develop a second tumor in the next 10 years. 22. MEN 1 (Wermer's syndrome) which is incorrect a. Autosomal dominant b. Background hyperplasia with the expected features in each gland. c. Pancreatic Islet cell lesions occur in 80% Page 34

d. Multicentric within each target organ. e. Pancreatic malignancy causes approximately 20% of deaths.

ABDO
1. Crohns, least likely A. colon ca B. sclerosing cholangitis C. hip arthropathy D. hydronephrosis E. sclerosing peritonitis 2. TOF, commonest type A. proximal atresia with distal TO fistula 3. Ulcerative Colitis patient with positive CEA suggests? A. possible colon Ca B. possible colon ca or acute exacerbation. 4. A 60-year-old male has colonic carcinoma, which involves muscularis mucosa. No regional lymph nodes, no distant metastasis. What is prognosis? A. 100% B. 95% C. 60% D. 40% Muscularis mucosa is superficial to muscularis propria, therefore Dukes <A / TisN0M0 / Stage 0 http://tinyurl.com/2ufbgx 5. Malignant gastric ulcer. What would be the indicator of malignancy? A. > 4cm (diameter) B. Single ulcer of greater curvature C. Heaped edges 6. Gastrinoma a.detect colonic hamartoma (?carcinoid?) 7. Patient has a peptic ulcer with increased serum gastrin levels, what would you scan for? d) Pancreas and duodenum e) Pancreas, duodenum and lymph nodes f) Mass in duodenum 8. With regards to carcinoid tumour, which is least correct? g) Equal incidence to small bowel adenocarcinoma h) Benign i) Yellow, tan, circumscribed j) Desmoplastic reaction h) Dec order of incidence appendix, SI, rectum Robbins p858

Page 35

9. Which of the following statements concerning gastrointestinal diseases is most correct? a) The colon is the most common site for GIT lymphoma b) Inflammatory pseudo-polyps are a characteristic feature of Crohn's colitis c) Apthous ulcers are a characteristic feature of ulcerative colitis d) Juvenile polyps typically occur in the rectum 10. HNPCC barium enema is performed to look for d) colonic carcinoma e) polyps f) detect colonic hamartoma 11. A 76 year old man, 5 cm thyroid mass, previously 2cm thyroid mass on imaging only 2 months earlier, when he had staging for an apparent Dukes B carcinoma of the caecum. The thyroid mass is most likely to be a) Papillary carcinoma of the thyroid b) Follicular carcinoma of the thyroid c) Anaplastic thyroid carcinoma d) Primary thyroid lymphoma e) Metastatic colon carcinoma 12. Benign gastric ulcer, ? which feature is most likely k) Greater curvature l) Heaped up edges m) Radiating folds n) Chronic atrophic gastritis 13. Coeliac, which is true a) Flattened mucosa, elongated crypts b) Less than 2 times the risk of cancer 14. Crohns, associated findings? a) PBC b) Renal stones c) Cancer 15. Gastric marginal zone lymphoma / MALToma. Which of the following statements is false? a) They are a form of B cell lymphoma b) They are most common in middle aged adults c) Early dissemination is typical d) They are associated with Helicobacter gastritis e) They are associated with chronic inflammation secondary to autoimmune disorders 16. Choledochal Cysts Most Correct a. presents 10-20s b. pain always means pancreatitis c. females 80% 17. Most/least likely wrt gastic carcinoma a. malignant tumour types in order of freq;; adeno, lymphoma, carcinoid and GIST b. alcohol a recognized risk factor c. macro excavated, exophytic, flat d. early gastric cancer confined to mucosa Page 36

18. CA 125 a. look for adenoca b. look for primary ovarian cancer c. look for primary colon cancer 19. where does HCC metastasize to most commonly? A. lung B. lymph nodes C. bone D. brain E. adrenal 20. Polyp question about hereditary nonpolyposis colorectal ca ? 21. you are ultrasounding a ?neonate/infant/child (?if sex mentioned) and the clinical hx is mother has Primary Bil cirrhosis. What is childs risk ? A. Same as for gen population B. 30 % risk according to most epid. Studies C. 50 % chance D. That of the gene frequency StatDx: Prevalence in families with one affected member is 1000 times higher than in general population. Prevalence: 19 to 151 cases per million population. Therefore: 19000 to 151,000 per million; = 2 15% 22. Primary Biliary cirrhosis involve: 23. most correct statement about chronic Cholecystitis is: 24. Coeliac disease most correct statement is a. definitive diag is by showing histologic total villous atrophy b. 10-20 % have anti gliadin antibodies without having the disease 25. Gastric CA could develop with a. pernicious anaemia b. H. Pylori infection 26. Pt with crohn's dx , 7cm green tumor in liver on biopsy a. cholangioca b. HCC 27. 32 female pt referred for a Ba enema H/O of HNPCC. Physician wants to a. rule out ca colon b. detect colonic harmatoma 28. adolescent female with pancreatic tumor a. papillary cystic tumor (=solid pseudopapillary tumor) 29. Epidemiology and incidence of hepatoma 30. What is not a complication of Gallstones 31. If ulcer on Ba Meal most import Hx to suggest Malignant ulcer Page 37

a. pernicious anemia b. crohn's c. Infection H. pylori 32.Dif erence between primary biliary cirrhosis and sclerosing cholangitis 33. Definition of pancreatic pseudocyst 34. Risk of Caecal carcinoma a) 1% synchronous b) 0.1% synchronous c) 1% synchronous and metachronous d) 39% ____ the caecum and ascending colon StatDx 5% synchronous; 5% metachronous. 35. What are risk factors for malignancy in a stomach ulcer a) pernicious anaemia b) Crohns disease c) Coeliac disease d) H.pylori 36. Crohn's disease in counselling a sibling of risk a) 1% b) no increased risk c) 10% 37. HNPCC - Ba enema is performed to look for (32yo female pt with Hx of HNPCC) a) colonic carcinoma b) polyps c) detect colonic hamartoma 38. Tracheo-oesophageal fistula - most common type? a) proximal oesophageal pouch and distal TOF 39. Gastrinoma. Patient diagnosed with Zollinger Ellison, most likely imaging findings include a) pancreatic or duodenal mass 40. Coeliac disease. What is the least likely? a) antigliadin Ab's b) treatment prevents development of lymphoma 41. Coeliac disease, most correct statement is a) definitive diagnosis is shown by showing histological total villous atrophy b) 10-20% have anti-gliadin Ab's without having the disease 42. HNPCC - ?incorrect option a) increase incidence endometrial ca in females b) earlier increase of colorectal ca than non-hereditary CRC c) Ca does not arise in pre existing polyps All correct

Page 38

43. 12 Polyps on colonoscopy of young male incorrect option a) >100 polyps of classic Dx of FAP b) PJ polyps only in small bowel c) Lack of FHx makes Dx less likely spontaneous mutations rare 44. AdenoCA oesophagus a. HNPCC b. H pylori 45. Nonspecific region of small bowel thickening on CT, young man, no Hx a. Previous Rx for seminoma b. RA new onset c. Rotavirus 46. Mid 1/3 oesophageal stricture 40yo female most likely a. If adeno Ca likely long segment Barretts b. SCC only <5% here c. SCC there are 4 patterns multifocal/luminal/ulcer/polypoid 47. A fifty year old female develops a colovaginal fistula. Ten years earlier she had treatment for choriocarcinoma and colonoscopy six months ago was normal. Most likely cause is a. Radiotherapy change b. Ca Colon c. Crohns d. Diverticular abscess e. Ca Bladder 48. Following not associated with increased risk of bowel malignancy a. Obesity b. Diabetes c. Crohns d. Meat/low fibre diet 49. Celiac disease associated with a. Less than 2x risk of malignancy b. Bowel wall thinning 50. Histologically appendicitis characterised by a. neutrophils in the mucosa b. neutrophils in the the muscularis c. Plasma cells/lymphocytes in muscularis d. serosal hyperaemia 51. Commonest site for adenocarcinoma colon commonest to rarest are a. Various orders of caecum, sigmoid, ascending, descending, transverse 52. Australian rugby player of the decade 1999-2009 a. Stephen Larkham b. Stirling Mortlock c. John Eales d. George Smith e. Tim Phillips Page 39

53. Carcinoma of the colon extends into the muscularis mucosa but not elsewhere. Prognosis WITH treatment is a. Approx 100% 5yr survival b. 95% 5yr survival c. 60% 5yr d. 40% 5yr e. 20% 5yr 54. Adenomatous polyps are NOT a feature of a. Gardners Syn b. Familial Polyposis c. Turcots Syn d. Peutz-Jeghers Syn e. Sessile Villous lesions 55. 5cm long distal oesophageal mass is biopsied yielding diagnosis of adenocarcinoma. Origin of lesion most likely a. Barretts oesophagus b. Extension from adjacent Ca lung c. Extension from fundal Ca stomach 56. Which of the following statements concerning gastrointestinal diseases is most correct? a) The colon is the most common site for GIT lymphoma (stomach) b) Inflammatory pseudo-polyps are a characteristic feature of Crohn's colitis (more common in UC) c) Apthous ulcers are a characteristic feature of ulcerative colitis (crohns) d) Juvenile polyps typically occur in the rectum 57. HNPCC barium enema is performed to look for a) colonic carcinoma b) polyps c) detect colonic hamartoma d) hereditary neoplastic colonic polyposis 58. Benign gastric ulcer, ? which feature is most likely a) Greater curvature b) Heaped up edges c) Radiating folds d) Chronic atrophic gastritis 59. Giardia Dx a. Stomach/proximal bowel b. Helminth c. Normal commensal in colon d. Common in Neonatal ICU 60. Alpha feta protein is most likely to be associated with a) colon carcinoma b) lung cancer c) pancreatic cancer d) smoking e) NSGCT tumour of testes Page 40

f) GIST g) Uncomplicated cirrhosis 61. CEA is least likely to be associated with a) cirrhosis b) hepatitis c) smoking d) alcoholic cirrhosis e) breast cancer f) pancreatic cancer g) RCC 62. Liver cirrhosis patient. History of ulcerative colitis (Chrons). Post mortem (Bx) showed a green mass 7cm size. What is the most likely cause? a) HCC b) Cholangiocarcinoma c) Adenoma d) Nodular regeneration 63. Risk of Caecal carcinoma a) 1% synchronous b) 0.1% synchronous c) 1% synchronous and metachronous d) 39% ____ the caecum and ascending colon e) 10% synchronous 64. What are risk factors for malignancy in a stomach ulcer e) pernicious anaemia f) Crohns disease g) Coeliac disease h) H.pylori 65. HNPCC - Ba enema is performed to look for (32yo female pt with Hx of HNPCC) d) colonic carcinoma e) polyps f) detect colonic hamartoma 66. The least likely association with Crohns a. Hypertrophic osteoarthropathy b. Ankylosing spondylitis c. Increased risk of colonic adenocarcinoma d. Primary biliary cirrhosis e. Erythema nodosum 67. The lowest association percentage wise in ulcerative colitis a. Colonic adenocarcinoma b. Toxic megacolon perforation c. Colonic strictures d. Terminal ileum involvement e. Arthropathy

Page 41

69. The following is least useful to differentiate between CROHN DISEASE and ULCERATIVE COLITIS; a. Depth of Ulcers b. Presence of crypt abcesses c. State of the Ileocecal valve d. Presence of Fistulae e. Eccentricity 70. The least accurate regarding Gastrointestinal Scleroderma is a. achalasia is characteristic b. gastroesophageal reflux is common c. "hidebound / accordion" pattern with small bowel involvement d. marked dilatation of proximal small simulating small bowel obstruction e. normal peristalsis above aortic arch 71. HELICOBACTER PYLORI INFECTION (incorrect) a. gram-negative spiral-shaped bacillus b. affects >50% of Americans >60 years of age c. asymptomatic in the vast majority d. increased risk of peptic ulcers, duodenal slightly more than gastric e. increased risk of diffuse type gastric carcinoma 72. The risk factors for gastric carcinoma except a. smoking & picked vegetables b. alcohol c. excessive salt d. pernicious anemia e. altered anatomy 73. A 20 year old has a lonstanding hard lump on his head and is diagnosed with a retroperitoneal desmoid tumour. His father and brother have multiple colonic polyps. The least likely association is a. Risk of periampullary and duodenal carcinoma b. Thyroid carcinoma c. Risk of malignant transformation of colonic polyp is 100% d. The other family member has familial polyposis coli e. Intestinal polyposis occurs earlier than extraintestinal manifestations 74. The incorrect statement regarding metastatic melanoma is a. Hematogenous dissemination to the small bowel is found less frequently than intraperitoneal seeding. b. Multiple, partly ulcerated antimesenteric nodules of submucosal origin are typical for hematogenous metastases. c. Common present with small bowel obstruction. d. Typical for melanoma is the non-obstructing, lumen filling intussusception. 75. The incorrect statement regarding carcinoids a. They are multiple in about less than 5% of cases. b. Of carcinoids that exceed 2 cm, 90% have already metastasized. c. Elevated 5-HIAA urinary excretion and decreased blood levels of tryptophan. d. The carcinoid syndrome of flushing, diarrhea and bronchospasm is a late development and is usually due to hormonally active liver metastases. Page 42

e. Carcinoid becomes a very likely diagnosis if there is contraction of folds at the tumor edge and/or if an outside mass is shown by increased separation from adjacent loops.

Hepatobiliary and Pancreas


1. CholangioCa mets various combinations of : a. Regional nodes b. Liver c. Lungs d. Bronchial tree e. Peritoneum, pericardium 2. 32 yo African immigrant: liver mass invading the IVC a. cholangioCa should be considered b. systemic leishmaniasis should be considered c. hepatoma atypical d. hepatoma unlikely in absence of aflatoxin e. hepatoma in absence of cirrhosis possible if early infection in childhood with HBV 3. 6cm mass, tail of pancreas and splenic hilum, coeliac nodes - ? Dx a. pancreatic ca b. lymphoma c. mucinous cystandeno ca d. serous cystadenoma 4. Causes of hepatic steatosis, except a. pregnancy b. obesity c. anoxia d. diabetes e. smoking 5. The following is a least likely association. a. VMA calcifying mass in a child b. CA125 endometrial carcinoma c. BHCG pt with previous hydatiform mole d. AFP liver mass in patients with cirrhosis e. Calcitonin pt with thyroid mass in MEN 2a 6. Primary Biliary cirrhosis involve: 7. Most correct statement about chronic cholecystitis 8. Raised Alpha Feto protein most likely with a. NSGCT of testes b. GIST c. uncomplicated cirrhosis d. colon CA e. lung CA 9. What is not a complication of Gallstones Page 43

10. Difference between primary biliary cirrhosis and sclerosing cholangitis 11. Definition of pancreatic pseudocyst

HAEM
1. What are likely findings on ultrasound on 16year old female with spherocytosis A. Gall stones. B. Renal stones. C. Splenomegaly D. Enlarged lymph nodes 2. What is the staging of Hodgkins lymphoma which involve cervical, mediastinal and crural nodes: A. Stage I B. Stage II C. Stage III D. IB E. IIS 3. The best diagnostic tests for multiple myeloma are a. Urinary BJP and serum electrophoresis 4. Staging of Lymphoma, Ca Lung, ,Neuroblastoma 5. A child with spherocytosis has abdominal pain best explained by a. Gallstones 6. Patient with Polycythemia rubra Vera, which is not true a. Transform to AML b. Transform to CML c. Develop Myelofibrosis d. Splenomegaly and cirrhosis e. Risk of stroke 7. In Thalassemia major, which is true a. Anaemia is secondary to abnormal b Hb b. Presentation age 6-9 months c. Splenomegaly only occurs if cirrhosis Thalassemia major = homozygous B-thal. Sickle cell (and Thal in clinical paeds texts, but no mention in Robbins) presents >5-6m age when HbF runs out; no mention of this phenomenon for Thal 8. Patient with abnormal bleeding, normal platelets but decreased factor VIII and cofactor ristocetin, diagnosis is A. Haemophilia A B. Haemophilia B-Christmas disease C. Von Willebrand disease D. Lupus anticoagulant E. DIC Page 44

Patients with von Willebrand disease have prolonged bleeding time despite a normal platelet count. The plasma level of active vWF, measured as the ristocetin cofactor activity, is reduced. Because vWF stabilizes factor VIII by binding to it, a deficiency of vWF gives rise to a secondary decrease in factor VIII levels. This may be reflected by a prolongation of the PTT in von Willebrand disease types 1 and 3. Robbins page 655 9. Patient with Hodgkins disease stage IIIES, which is not true A. Nodes above diaphragm B. Nodes below diaphragm C. Involvement of extra nodal sites D. Involvement of spleen E. Involvement of liver 10. Patient with Waldenstroms macroglobulinemia A. Bone lesions indistinguishable from multiple myeloma B. Bone lesions in <3% C. Bone lesions involving cortex D. Solitary bone lesion most likely 11. DECREASED FACTOR VIII E. Christmas disease F. Haemophilia A G. Haemophilia B H. Von Willebrands 12. POLYCYTHAEMIA RUBRA VERA LEAST LIKELY CHANGES I. transform to AML J. transforms to CML K. associated with hepatic cirrhosis L. assoc. with Budd- Chiari M. assoc. with myelofibrosis 13A. Follicular lymphoma - incorrect a. Common 15-40 year olds b. Nodular growth, follicular B cells c. T(14;18) translocation d. Transformation occurs to diffuse large B-cell lymphoma in 30 50%, rarely to aggressive form of lymphoblastic lymphoma or leukaemia. e. Is characterised by cells in stroma that can change to diffuse appearance 13B. Follicular lymphoma - incorrect a. low grade on old working classification b. Is indolent but incurable c. Good survival, 7-9 years. d. Poor chemo response. e. commonly arises in gut 14. Regarding lymphoma A. Lymphocyte depleted is most common B. RS cell is derived from B cell C. HD and NHD can coexist D. FNA cannot Dx HD as it is the architecture of lymph node that makes the Dx ?? Page 45

(c) can be argued, given that HD can transform into DLBCL, but (b) is most correct 15. Hodgkins, false? i. Common in waldeyers 16. Downs syndrome gets anemia and hepatosplemegaly. Suspect a) CLL b) ALL, AML C) CCF 17. Hodgkin Disease, 50yo, with Chemo, develops lung parenchymal mass. a. CMV b. HD recurrence c. Primary TB d. Secondary TB Still cant decide on this one who the f*** knows whether hes more likely to get TB of parenchymal HD 18. Beta thalassemia a. Splenomegaly b. Liver disease c. Transfusion disease 19. CLL and Small Cell lymphoma a. Same thing b. Dedifferentiated/transformed 20. Favour HD vs NHL a. More in males b. Regional nodes c. Abdominal involvement (NHL) 21. Pt with CML, distal narrowing of the oesophagus a. graft vrs host b. CML 22. Monoclonal gammopathy need radiology a. CT Chest/Abdo/Pelvis b. CT Head/Neck/C/A/P c. Plain films axial skeleton & proximal limbs d. Plain films axial + hands 23. Least common and most controversial type of Hodgkin lymphoma A. Lymphocyte predominant B. Lymphocyte rich C. Nodular sclerosing D. Mixed Cellularity E. Lymphocyte depleted 24. Hodgkins and staging of IIIE or IIIES.. least likely (??) A. Nodes only above diaphragm B. Nodal and extranodal below diaphragm Page 46

C. Involves/doesnt involve liver 25. B thal major, most likely ? A. presentation at 6-9/12 26. A 24 yr old woman with Stage II lymphocyte predominate Hodgkin's lymphoma has been neutropaenic for 2 weeks in intensive care. Since her pre-treatment CT scan 3 months earlier her liver and spleen have increased in size and show new lesions up to 2 cm in size. Despite multiple antibiotics-she continues to decline and CT shows a new left MCA infarct. This is most compatible with; a) Systemic candidiasis b) Bacterial endocarditis with systemic emboli /infarction c) Drug resistant lymphocyte predominate Hodgkin's lymphoma d) Transformation to a more aggressive lymphoma e) Non-bacteria thrombotic endocarditis with systemic emboli / infarction 27. Hospital pathology computer shows that the serum electrophoretic analysis and urinary Bence Jones protein assessment are normal. Which of the following statements is most correct? a) The finding excludes multiple myeloma b) 1 in 100 cases of multiple myeloma would fit this pattern. c) 20% of cases of multiple myeloma would fit this pattern. d) Bence Jones protein assessment should have done on serum; in approximately 70% of patients the electrophoretic pattern alone is normal 28. gastric marginal zone lymphoma / MALToma. Which of the following statements is false? a) They are a form of B cell lymphoma b) They are most common in middle aged adults c) Early dissemination is typical d) They are associated with Helicobacter gastritis e) They are associated with chronic inflammation secondary to autoimmune disorders 29. Difference between Stage Ia and Ib of Hodgkins lymphoma include; a. All stages are further divided by absence (A) or presence (B) of the following systemic symptoms: significant fever, night sweats, and/or unexplained weight loss of greater than 10% of normal body weight. b. 1a is more common in Africans, 1b in males named Bob. c. 1b is derived from the japanese term wun-be describing disease fevers and medistinal masses. d. 1a is a mistranslation due to queensland hematologists describing the condition as stage 1.. ay. 30. Pt with Macrocystic anemia most likely cause is : a. celiac disease b. cystic fibrosis c. gastritis d. Crohn's

MSK
1. AVN of hip. Alternative diagnosis should be considered if: a) HLA B27 positive b) Deep sea diver c) Pulmonary eosinophilia Page 47

2. RE Reiters disease, which is true a. occurs after N gonorrhoea urethritis b. Shigella bowel infection 3. Pagets disease: Which is true? a) less likely if Japanese tourist. b) Causes cranial nerve palsy 4. Patient thought to have multiple myeloma, but negative serum electrophoresis. a. if Bence Jones is negative, this can occur in 1% of patients with MM b. not myeloma, but Waldenstroms macroglobinaemia 5. Ewings sarcoma versus osteomyelitis, more likely Ewings if a. Fever b. Afro-American c. Involves ilium 6. Patient with Waldenstroms macroglobulinemia a. Bone lesions indistinguishable from multiple myeloma b. Bone lesions in <3% c. Bone lesions involving cortex d. Solitary bone lesion most likely 7. Solitary Plasmocytoma of bones of nasal cavity, which is true a. Most go onto multiple myeloma (extraossesous rarely progress to mm) b. Something correct hopefully. 8. Osteoporosis, which is most true a. Decreased bone density b. Senile osteoporosis, increased osteoclast activity c. Treated with diphosphonate to stimulate osteoblasts d. Delayed fracture healing with poor callus formation e. Decreased serum calcium and phosphate 9. Chondrosarcomas a. Arise in same area as enchondromas b. Low-grade tumours have good prognosis c. Chondroblastic osteosarcoma is recognised type d. Occur 15-40 year olds 10. The feature most diagnostic of gout is a. Acute pain 1st MTP joint b. -ve birefringent crystal on aspirate c. para articular erosions d. hyperuricemia 11. CPPD incorrect a. Hypo Mg b. RA c. OA d. Hemochromatosis Page 48

12. PVNS incorrect a. ankle commonest b. not neoplastic c. causes erosions 13. Chondrocalcinosis is not found in a. Hemochromatosis b. RA c. Hypomagnesemia 14. Sclerotic lesion thought to be Pagets. Least likely f. Japanese tourist g. Female h. lytic lesion in same bone i. cranial nerve palsies 15. Patient with AVN hip. Least likely cause a. Anoxia b. Abalone diver c. SLE d. Pancreatitis 16. LCH a. Letterer Siwe mostly results in punched out skull lesion b. Letterer Siwe 2-7 yo c. teenager, can present in skull d. Most frequent to least, tibia to skull 17. Osteosarcoma definition a. A malignant mesenchymal tumor which produces bone matrix or osteoid. b. An unconscious sarcastic osteopath. 18. Extra osseus osteosarcoma versus myositis ossificans a. Pathologists cant work it out so dont biopsy MO. 19. Liposarcoma. Most correct statement a) occurs in retroperitoneum and proximal limbs b) 2-8 years of age c) 50-80 years of age 20. Which of the following statements concerning paget's disease of bone ls least correct? a. There is an initial osteolytic phase b. current evidence suggests it is due to a slow virus infection of bone c. Involvement of the ribs, fibula or small bones or the hands or feet is unusual d. Recognised complications include malignant fibrous histiocytoma and chondrosarcoma e. the overall lamellar pattern remains ordered in the absence of malignancy 21. Osteoblastoma v Osteoma a. No such thing as an osteoblastoma b. spinal involvement

Page 49

22. Most correct about chondrosarcoma a. chicken wire calcification b. extension in bone marrow c. involve lower extremities 23. most correct statement about synovial sarcoma a. to regional lymph nodes, lung and bones b. 3rd 5th decade 24. Girl presents with precocious puberty and expansile lesions in pelvis and proximal femur. Which other bones are likely to be involved? a. Ribs, skull, jaw b. Tibia, skull, spine c. Spine, wrist 25. Chondrosarcomas ? a. chondroblastoma may give rise to chondrosarcoma b. chondroblastic osteosarcoma is part of chondrosarcoma classification c. Occur 15-40 years 26. Ewings tumour, least likely? A. involves ilium (? Whether spelling mistake) B. Found in blacks. 27. Histology of non-ossifying fibroma a. Consists of unossified cartilage b. Consists of acellular fibrous material c. whorled bundles of spindle-shaped fibroblasts + scattered multinucleated giant cells + foamy xanthomatous cells 28. Most correct statement about Paget's disease a. Histological hallmark is woven Bone b. Large giant osteoclasts, polynuclear c. Slow rotavirus infection 29. Osteoblastoma vs Osteoma a. No such thing as an osteoblastoma b. spinal involvement 30. most correct statement about synovial sarcoma a. mets to regional lymph nodes, lung and bones 31. Chondrosarcoma - differentiating from other chondroid tumours a) Peripheral skeleton b) Intramedullary chondroid matrix surrounding trabeculae c) Cellular atypia d) Chicken wire appearance 32. Child with mass FNA shows small round blue cells- least likely diagnosis is C. Neuroblastoma D. Ewings sarcoma E. Rhabdomyosarcoma Page 50

F. Wilms tumour G. Retinoblastoma 33. SCLERODERMA AND HAND CHANGES a. 24yo female and Raynauds phenom b.54yo female and Raynauds phenom c. male and Sjogrens 34. The best diagnostic tests for multiple myeloma are a. Urinary BJP and serum electrophoresis b. tests that look for not just one but several myelomas 35. Chondrosarcoma - differentiating from other chondroid tumours f) Peripheral skeleton g) Intramedullary chondroid matrix surrounding trabeculae h) Cellular atypia i) Chicken wire appearance 36. Neuroblastoma, prognosis : a) n-myc amplification 37. Fatty lesion next to effaced kidney. Liposarcoma? Which of the following is most relevant? a) Tuberous sclerosis b) Previous radiotherapy c) The patients name is Marjorie Dawes. 38. Least likely bone aneurysm f) Marfans g) NF1 h) SLE 39. Scleroderma least correct a. Compared to RA more hip destruction b. CNS kidney involved c. Esophagus fluid level d. Basal lung fibrosis 40. 50 yo male, mass 6cm, near knee but not part of bone with Ca2+, ? synovial sarcoma i. Hx c/w synovial sarcoma j. Unlikely due to age/sex k. Unlikely due to clinical Hx l. Unlikely due to position m. Unlikely due to something else 41. Calcium pyrophosphate dihydrate crystal deposition disease A. disproportionate narrowing of patellofemoral joint B. involvement of tendons, bursae, pinnae of the ear C. resembles osteoarthritis D. large subchondral cyst would be an atypical finding E. numerous intra-articular bodies

Page 51

42. Subchondral cyst may be present in all of the following except: A. osteoarthritis B. rheumatoid arthritis C. osteonecrosis D. cppd E. hemophilia 43. Engelmann-camurati disease, the least accurate statement A. may initially have amorphous increase in density at base of skull B. fusiform enlargement of diaphyses with cortical thickening & progressive obliteration of medullary cavity C. bilateral symmetrical distribution involvement with progression of lesions along long axis of bone toward either end D. metaphyses and epiphyses often involved E. relative elongation of extremities 44. The following are associated with chromosomal abnormalities except A. Liposarcoma B. Synovial sarcoma C. Rhabdomyosarcoma D. MFH E. PNET 45. Chondrosarcoma may arise from all except A. Enchondroma B. Osteochondroma C. Pagets D. Fibrous dysplasia E. Chondromyxoid fibroma 46. Chondrosarcoma incorrect A. half as common as osteosarcoma B. peak age 40-70yrs C. more common in the central skeleton D. 10% of low grade lesion dedifferentiate into more high grade lesions E. 5 year survival for a grade 3 is <10% 47. Benign fibrous histiocytoma may be differentiated from non-ossifying fibroma by the following except A. Older age B. Central location C. Usually symptomatic D. Haemosiderin pigment E. Presents with painful soft tissue swelling 48. According to the Tang Rule of 50 for Chordomas, the least correct is: A. 50% calcify B. 50% sacrococcygeal C. 50% of vertebral involvement is in the cervical spine D. Less than 50% of spheno-occipital lesion has a chondroid component E. Metastasizes in more than 50%

Page 52

49. According to the Tang rule of 50 for MEN 2, the following are true except: A. Phaechromocytomas are frequently bilateral B. Medullary thyroid carcinoma present in 50% C. Phaechromocytomas are present in approximately 50% of patients D. Post adrenalectomy 50% will develop a second tumor in the next 10 years. E. In MEN 3, 50% cases are sporadic 50. Osteoarthritis - incorrect a. In the primary form effects middle aged women almost exclusively b. Involving the hip, the maximal joint space narrowing is found superiorly c. Erosive form is also known as Kellgrens arthritis, and is associated with severe osteoporosis and gullwing pattern of erosions d. Gas in the apophyseal joint is pathognomonic of the degenerative process 51. With regards to rheumatoid arthritis a. In the hands it is always symmetrical bilaterally b. May demonstrate a high riding shoulder, which is defined as greater than 3 cm distance between the humerus and acromion c. Hip migrates superiorly, as opposed to the axial migration in osteoarthritis d. Erosion of the ulnar styloid and triquetrum is characteristic 52. Ankylosing spondylitis incorrect: a. More common in males b. HLA B27 positive in 95% c. Is characterized by vertebral body squaring and flowing osteophytes d. Enthesopathy is common 53. Reiters syndrome - incorrect a. More common in males b. Follows nongonococcal urethritis or bacterial dysentery c. Commonly demonstrates the classic triad of urethritis(or cervicitis), conjunctivitis and arthritis d. Radiologic features include; proliferative erosions of the MTP, retrocalcaneal bursitis and enthesopathy at the Achilles tendon insertion. 54. Regarding crystal deposition arthropathy a. Gout is characterized by marginal paraarticular erosions with overhanging and sclerotic edges, with preserved joint space. b. The main radiographic features of CPPD are chondrocalcinosis and arthropathy resembling osteoarthritis c. CPPD commonly involves the MCP, patellofemoral and radiocarpal joints d. Chronic haemophilia may be characterized by epiphyseal overgrowth, gracile diaphysis and widened intercondylar notch. e. Haemophilia may be similar radiographically as JRA.

Paediatrics
1. Downs syndrome gets anemia and hepatosplemegaly. Suspect? c) CLL d) ALL

Page 53

2. Patient with possible ADEM. Which clinical setting least likely? a. 2 week history of viral illness b. female c. headache and confusion progressing to coma in 48 hrs d. HIV positive e. Age less than 40 3. 16 year old with Freidrichs ataxia has a poor quality MRI. Most likely cause is? j. intention tremor k. recurrent facial tic l. orthopnoea m. salaam spasms n. hemiballismus 4. NF1 Which is not/least associated? o. Wilms p. Rhabdomyosarcoma q. CML r. phaeochromocytoma s. osteosarcoma 5. Newborn with multiple small renal cysts. Mother, father and sister to be imaged. Which is most likely? t. Mother, father and sister have normal imaging u. M & F normal, sister may be abnormal v. Sibling may have it w. Both parents may have it 6. LCH a. Lettere Siwe mostly results in punched out skull lesion b. Lettere Siwe 2-7 yo c. in a teenager, can present in skull d. Most frequent to least, tibia to skull. 7. 8yo T21 develops anaemia + splenomegaly + thrombocytopenia. Likely cause? a. right heart failure b. CML c. Acute leukaemia 8. EBV manifestions and associations 9. Premature neonate and complications a. NEC b. Intracranial haemorrhage c. Lung microscopic findings 10. Cavernous lymphangioma in neck of child

11. Cryptorchism A. Assoc with inguinal hernias Page 54

B. Malignancy C. Infertility 12. Least likely to have 3 cells of origin A. Neonate with sacrococcygeal teratoma B. Female with benign teratoma C. Testicular tumour 13. 6 week old child on ultrasound has enlarged kidneys with multiple 1-2mm cysts throughout the kidney. Also has moderate renal / mild hepatic impairment. U/S of the parents and 2 yr old asymptomatic sister's kidneys? a. Parents normal, sister involved b. Father has "forme fruste" form of the condition c. Parents and sister normal d. (something about autosomal dominant form of the condition). 14. Haemolytic Uraemic Syndrome? a. Due to infection with Rotavirus b. Due to Shiga like toxin. E. Coli, Salmonela and Shigella 15. Which is false regarding nephroblastomatosis? a. resolve by 1 year of age b. precursor to Wilms disease c. include nephrogenic rests and nephroblastomas? in adult population d. renal hamartoma 16. Cryptorchidism a. Most commonly bilateral b. Is associated with infertility even after orchidoplexy c. Is not associated with inguinal hernia as canal is not formed 17. Which of the following is most correct regarding fibrous dysplasia? a. histology shows a "mosaic" appearance b. no malignant potential c. monostotic converts to polyostotic in 10-1 5% d. McCune-Albright represents polyostotic fibrous dysplasia, caf6-aulait spots with endocrine abnormalities (hyperthyroidism, gigantism, and iso-sexual precocity) e. most common locations include tibia > phalanges> femur> ribs > calvarium 18. Langerhans Cell Histiocytosis, most likely? a. Letterer-Siwe is dominated by "punched" out lesions of the calvarium b. Letterer-Siwe most common in 2-7 age group c. Can solely involve the skin or lungs in children or adolescents d. Does not affect the posterior pituitary e. None of these seem likely 19. Pt has neurofibromatosis type 1, which tumour is not associated with it? a. Phaeochromocytoma b. Chronic myeloid leukaemia c. Carcinoid tumour d. Medullary carcinoma of thyroid Page 55

e. Osteosarcoma 20. Pt with MEN 1, which is unlikely? A. Hypercalcaemia B. VMA in serum C. increased ALP D. elevated prolactin 21. MEN 1 features - incorrect a. increased PRL b. increased gastrin / gastrinoma c. hyperparathyroidism d. phaechromocytoma 22. 8yr old trisomy 21 patient with 1 month history of anaemia and splenomegaly. Which is true? a. ALL b. CCF + congestive changes 23. Infant born premature. What wouldnt you see? A. cerebellar haemorrhage B. retinal blindness C. NEC and strictures D. PDA E. changes which sounded like BPD lung 24. Staging of Neuroblastoma (tumor crosses midline, calcified) n. Stage II o. Stage III p. Wilms q. Need age to stage ? 20. Child Head trauma, suspected hematoma near pituitary stalk, diabetes insipidus a. Dilute urine, high serum osmo b. Concentrated urine, elevated serum osmo c. Post pituitary destruction d. Not DI but infact some other process eg LCH 21. Kidney, 1yo with 3cm mass removed, pathology says nephrogenic rests, most correct a. Not a real entity b. Increased risk of Wilms c. Possible entity but not in this age group d. Only micro Dx 22. CF a. Late onset recognized as chronic pancreatitis. b. If significant liver disease, consider alternative Dx c. Mg co-factor for Cl channel d. ?another liver stem. 23. Giardia Dx a. Stomach/proximal bowel b. Helminth Page 56

c. Normal commensal in colon d. Common in Neonatal ICU 24. EBV least true t. Meningoencephalitis u. Periventricular Ca2+ v. Pharyngitis w. Hepatitis 25. Children with ependymoma a. Prognosis poor cannot remove completely b. Less common than medullo c. Does not metastaize d. Myxopapillary variant in filum terminale e. 4 yr survival with complete resection 26.15 yo with psychosis, MRI showed diffuse WM abnormality a. Metachromic Leukodystrophy b. ADEM c. Huntingtons d. HIV 27. Huntingtons chorea what does it show a. Various locations including basal ganglia b. Caudate nuc +/- putamen c. Cerebellum d. Locus Cereus? 28. Downs Syndrome dementia most correct a. plaques b. amyloid c. accelerated deposition of plaques/tangles 29. Incidence of polycystic kidneys e. 1:1000 f. 1:10000 g. 1:100000 30. Following NOT associated with increased risk of testicular malignancy h. Cryptorchidism i. Testicular feminisation j. Testicular microlithiasis k. Klinefelters syndrome l. TB of the testis 31. Neuroblastoma worse prognosis a N-myc amplification 32. Which of the following is not a congenital CNS infection o. chickenpox p. rubella q. CMV Page 57

r. Toxoplasmosis s. Herpes 33. With regards to CMV, which is true? a. Accompanied by PCP in the lung b. >90% morbidity in infants CMV and PCP are common causes of diffuse infiltrates in lung in IS patients. In AIDS, PCP shows up at <200 CD4 while CMV needs <50 CD4. Has high prevalence with Kaposi sarcoma in AIDS (also <50 CD4) 34. With regards to cystic fibrosis, which is false? a. Increased incidence of adrenal haemorrhage b. intussusception c. involves the salivary glands 35. Kleinfelters, which is true? a. XYY b. 1 : 10 000 c. recognized presentation is gynaecomastia in adolescence d. diagnosed by characteristic facies in neonate gynaecomastia, paucity of hair, rarely dx b4 puberty 36. FNA small blue cells a. Wilms b. Ewings c. Rhabdo d. Neuroblastoma e. Also lymphoma 37. You are reviewing a case of Non accidental injury, looking at records of a sibling, the most striking finding is: a. SIDs of a sibling 13 months old b. Healing fracture of the clavicle at 1 month c. Brachial Plexus injury 38. Chlamydia pneumonia 39. The main abnormality in Cystic fibrosis is : a. Transmembrane electrolyte transport abnormality 40. Wilm' s tumor age presentation a. 2-5 yrs 41. Adolescent female with pancreatic tumor a. papillary cystic tumor 42. Child with posterior paravertebral mass, biopsy shows mature neural elements and Schwann cells not attached to nerve diagnosis is H. Ganglioneuroma I. Ganglioglioma J. Neurofibroma K. Schwannoma Page 58

L. Neuroblastoma 43. Child with mass FNA shows small round blue cells- least likely diagnosis is M. Neuroblastoma N. Ewings sarcoma O. Rhabdomyosarcoma P. Wilms tumour Q. Retinoblastoma 44. Patient with cystic fibrosis- long term survival is associated with R. No increased risk of malignancy S. Increased risk of malignancy only if has transplant T. Increased risk of GIT and lymphoma U. Increased risk of GIT, pancreas, biliary and lymphoma V. Increased risk of cholangiocarcinoma 45. Trisomy 21- which is true a.. 0.5% are mosaic/ translocations b. 5% have associated CHD c. Live <30 years d. If live > 40 years 80% have Alzheimers type neurodegenerative disorder 46. Tracheo-oesophageal fistula, most common type is a. Proximal fistula, distal blind end pouch b. H type c. Distal fistula, proximal pouch d. Misnomer because no atresia e. Misnomer because no fistula 47. TRISOMY 21 MOST CORRECT a. increased risk of CML b. greater than 80% Alzheimers type change over 40yo c. mosaicism <0.1% (1% are mosaic) 48. Patient with webbed neck, coarctation and thyroid disease a. Turners with toxic adenoma b. Turners with Hashimotos c. Turners with hyperfunctioning adenoma 49. Child for Ultrasound whose mother has PBC. What is the likelihood of PBC in child: a. same as general population b. 20% c. 50% d. increased depending on gene frequency e. 30 % risk according to most epid. Studies StatDx: Prevalence in families with one affected member is 1000 times higher than in general population. Prevalence: 19 to 151 cases per million population. Therefore: 19000 to 151,000 per million; = 2 15% 50. Neoplasms assoc with CF ? A. Only with transplant pts in assoc with immunosupp B. Increase in pulm and abdom lymphoma Page 59

C. Increase in pulm, hepatic and panc tumours 51. Wilms tumour overall survival ? A.40-50 % B. 80-90% 52. Downs syndrome.. most likely? A. 5% risk of CHD B. 80% Alzheimer type dementia after age 40 C. 1 % due to ? mosaic/ ? translocation E. 20-30 % survive past 30 53. Young girl with moderate to severe sarcoid presents with sudden death cause? F. Renal failure G. Pulmonary HT H. Ruptured berry aneurysm I. Hypertensive cerebral haemorrhage J. Cardiac sarcoid 54. A 6year old child has large mass involving adrenal gland/kidney and one ipsilateral lymph node, urine is positive for VMA. What is the staging? A. Stage I B. Stage II C. Level 2 D. Stage II S E. Level 3 55. Girl presents with precocious puberty and expansile lesions in pelvis and proximal femur. Which other bones are likely to be involved? A. Ribs, skull, jaw B. Tibia, skull, spine C. Spine, wrist 56. Kawasakis disease is A. A self-limiting disorder with less than 1% death rate B. Associated with a 20% death rate C. Characterised by recurrent attacks 57. A 16yr old female 140cm in height and weight 57kg has primary amenorrhoea. Most likely cause A. Testicular feminisation B. Prolactinoma C. Anorexia D. Polycystic ovaries E. Turners syndrome 58. A child with spherocytosis has abdominal pain best explained by A. -Gallstones 59. 8yr old girl having anaesthetic for MRI to investigate choreiform movements. Murmur heard by anaesthetist most likely due to Page 60

a. Previous rheumatic fever 60. Features of cystic lymphangioma do NOT include a. -axillary location b. -child 18 months old c. -large cystic spaces 61. 8yr old girl having anaesthetic for MRI to investigate choreiform movements. Murmur heard by anaesthetist most likely due to a. Previous rheumatic fever 62. Kawasakis disease is B. -A self-limiting disorder with less than 1% death rate C. -Associated with a 20% death rate D. -Characterised by recurrent attacks 63. Features of cystic lymphangioma do NOT include a. -axillary location b. -child 18 months old c. -large cystic spaces All true 64. B thal major, most likely ? A. presentation at 6-9/12 65. Fragile X a) mental retardation b) mandible long c) hyperextensible lax joints All true 66. CMV encephalitis a. Characteristic inclusions b. Ependymal and subependymal spread c. May cause haemorrhage

BREAST
1. Pagets of the nipple - reasons why you can't see it on mammo a. Involves skin of areola and rarely extends further back than lactiferous sinuses b. subareolar DCIS not easily identifiable on mammogram 2. Colloid tumour of the breast is usually a rounded well defined lesion. Most likely because a. swimming in a pool of mucus b. cells in indian file c. signet ring cells d. intense lymphocytic infiltrate and no desmoplastic response e. if granulomas are present this implies cant be infection or malignancy 3. An area of microcalcification was biopsied and revealed LCIS. Which is most likely regarding the calcification? Page 61

a. Due to necrotic debris b. Due to mucin c. Incidental LCIS doesnt calcify 4. Breast calcification biopsied path report LCIS likely explanation: a) Ca++ due to necrotic cells in duct lumen b) Sclerosis / fibrosis / inflammation around duct c) LCIS does not Ca++ and is incidental finding d) Ca++ implies invasion of the stroma 5. Incidence of breast Ca not associated with a family Hx. Which is most correct? a. 90 - 95% b. 80- 85% c. 50 - 70% d. 20-25% e. excluding those with the BRCA gene, 95% cases are nonfamilial 6. What is the percentage of breast Ca with no past family hx b) 95-97% c) 80- 85% d) 50-75% e) 20-30% f) greater than 95% in absence of BRCA gene 7. Phylloides a. 30-50% will have LN mets at diagnosis b. same age as fibroadenoma c. low grade lesions difficult to distinguish histologically from fibroadenoma 8. Fibroadenomas a. Can increase in size with lactation and infarction b. Usually less than 35y c. In patients on cyclosporin 9. Breast cancer, 1.5 cm, enlarged mobile lymph nodes, which stage? a. Stage I b. Stage II c. Stage III d. T1 or T2 depending on biopsy of lymph node 10. BRCA 1, BRCA 2, which is false? a. Both increase risk of ovarian cancer b. Increase risk of prostate cancer in both c. 50-70/80% in those with fam hx ?breast ca 11.Sclerosing adenosis, which is not true? a. Distinguishable from cancer as it doesnt cause compression 12.Lobular cancer, why cant you see it? a. Indian file

Page 62

13. Breast cancer which type has BEST prognosis / histological prognostic factors for breast cancer ? a Tubular 14. Types of breast carcinoma-in-situ: which is NOT? a. Cribriform b. Papillary c. Micropapillary d. Comedocarcinoma e. Medullary 15. Breast which is not a proliferative disease a. Sclerosing adenosis b. Periductal mastitis 16. Which does not follow DCIS a. Invasive duct b. Invasive lobule and duct c. Medullary d. Tubular e. Colloid 17.Regarding Ca2+ which is true a. Breast cysts have curvilinear Ca2+ b. MicroCa2+ + phyllodes c. MicroCa2+ and medullary Ca2+ d. Sclerosing adenosis 18. Re Phylloides tumour, which is true a) neoplastic stromal and ductal component b) same age affected as FA c) resemble FA histologically when low gd d) assoc with cyclosporin 19.Pagets disease of the nipple with no mammo abnormality because: a) restricted to areola, very rarely (<1%) extends into lactiferous ducts b) eczematoid reaction c) associated with LCIS d) associated with DCIS e) Involves skin of areola and rarely extends further back than lactiferous sinuses f) subareolar DCIS not easily identifiable on mammogram g) More commonly associated with lobular carcinoma 24. A breast papilloma is least likely to be a. subareolar b. 3cm c. frond like on histology 25. LCIS, which is true a. Develop lobular carcinoma at same site b. Increased risk of lobular carcinoma bilateral c. Increased bilateral risk of ductal and lobular Page 63

26. Regarding breast which is true a. Fibroadenoma associated with increased risk of cancer b. Phyllodes tumour occurs in same age group as fibroadenoma c. Fibroadenoma occur in patient on cyclosporin d. Polyclonal or monoclonal proliferations 27.Breast cancer, which type has worst prognosis a. Lobular b. Ductal, NOS c. Medullary d. Tubular e. Papillary 28. Breast ca type with worst prognosis ? A. invasive ductal Ca B. invasive lobular Ca 29. Well defined mass a. DCIS b. LCIS c. Medullary d. Tubular e. Invasive 30. LCIS in 2003 a. Bilateral breast increase risk b. Increase risk in that breast 31. DCIS removed in 2003 a. 10-20% DCIS risk b. 1% die of disease c. Mastectomy = local conservative surgery outcome/?mortality? 33.LCIS a. increased incidence of L-Ca b. increased incidence of L-Ca and ductal Ca 34. Patient with BRCA1 and known Ca breast aged 70yrs. Risks of Ca Ovary and type are a. 4-9% No particular type b. 15-20% Cystadenocarcinoma c. Various risks and subtypes. BRCA associated with epithelial ovarian tumours. 35.A new breast screening technique detects 84% ductal, 7% lobular and 2-3% medullary carcinoma. We can conclude a. New technique insensitive to lobular Ca b. New technique cannot detect Ca in situ c. lobular is 10% of all cancers. 36. Causes of nipple discharge in order of frequency (most common 1st) a. Duct ectasia, Papilloma, Intraductal carcinoma b. Duct ectasia, Intraductal carcinoma, Papilloma Page 64

c. Papilloma, Duct ectasia, Intraductal carcinoma d. Papilloma, Intraductal carcinoma, Duct ectasia e. Intraductal carcinoma, Duct ectasia, Papilloma 37. Risk factors for breast carcinoma, except a. Family history b. Early menarche, late menopause c. Late First pregnancy d. Multiparity e. Lobular carcinoma in situ (LCIS) is not in itself considered malignant but carries a 30% risk of breast cancer (15% in each breast)

MISC
1. The pathogenic agent of Legionnaires disease is L. pneumophilia. This agent is best described as a? a. gram negative bacteria b. spore forming coccus c. helminth d. protozoa e. richettsia 2. Amyloidosis which is least likely involved a. CF b. renal failure c. myeloma d. RA e. CML f. ankylosing spondylitis 3. 25% people who have required resuscitation after a contrast reaction dont react the 2nd time why? a. Type 1 reaction b. Type 2 reaction c. Type 3 reaction d. Type 4 reaction e. Atypical for a hypersensitivity reaction or not classical allergic reaction 4. Which is unlikely to be elevated in MEN I syndrome? a. Alk Phos b. Gastrin c. Ca d. Prolactin e. VMA 5. Recent decrease in incidence of Kaposis Sarcoma. ?Main reason for this a. Decreased incidence EBV b. Increased proportion of heterosexuals getting AIDS c. HAART retroviral treatment d. community education Page 65

e. prophylactic treatment 6. Least likely to have 3 cells of origin a. Neonate with sacrococcygeal teratoma b. Female with benign teratoma c. Testicular tumour 7. Regarding Mycobacterium tuberculosis, which of the following is least correct? a. Risk factors include HIV, chronic renal failure b. Mantoux test does not differentiate between active disease and previous exposure c. Mantoux test may be negative in severe active disease d. 1: 1 0 000 develop active disease e. in immunosuppressed individuals, disease looks like progressive primary disease, involving middle to lower lobes 8. Which is least correct regarding Cryptococcus and its manifestations? a. basal ganglia lesions b. basal meningitis c. lung changes ?focal consolidation d. sinusitis 9. Fat embolism - which is false? a. 80-90% are asymptomatic b. can present with confusion, ataxia and irritability and coma c. 20-50% petechial rash d. SOB/ tachypnoea symptoms develop within 6 hrs e. Bleeding ?described site of bleeding ?cerebral haemorrhage 10. Hemochromatosis least correct a. Homozygous recessive b. Iron is directly toxic to host tissues c. Cardiac arrhythmia cause of sudden death d. Incidence of hepatoma is recessive e. Patients with longstanding disease develop cirrhosis + hyperbilirubinemia 11. Immune reaction to tuberculosis is an example of a. Antibody mediated immunity b. Delayed hypersensitivity reaction c. Immune complex mediated d. Immunosuppression 12. Regarding necrosis (cell death). Which of the following is MOST CORRECT: a. Liquefaction necrosis is characteristic of ischaemic destruction of cardiac muscle b. Councilman bodies in the liver in toxic or viral hepatitis is an example of apoptosis . c. The dead cell usually shows decreased eosinophilia. d. Caseous necrosis is encountered principally in the centre of the Aschoff nodule. e. Expansion of the nucleus of dead cells with unravelling of the chromatin is called pyknosis. 13. Regarding hemochromatosis which is least correct a. autosomal recessive b. less common in females. c. accumulation is life long. Page 66

d. effects are due to direct toxic effect of iron on cells. e. end stage cirrhosis and hyperbilirubinemia 14. with regards to CMV, which is true? a. Accompanied by PCP in the lung b. >90% morbidity in infants CMV and PCP are common causes of diffuse infiltrates in lung in IS patients. In AIDS, PCP shows up at <200 CD4 while CMV needs <50 CD4. Has high prevalence with Kaposi sarcoma in AIDS (also <50 CD4) 15. Patient hep D positive, needle stick injury to staff. Which is true a. No risk because staff member doesnt have hep B b. No such thing as hep D c. More at risk of hep B 16. Cystercercosis a. involve heart b. calcifies as it is inactivated or dies 17. Atypical Scenario a. Craniopharyngioma in a 42 year old b. Anaplastic thyroid cancer in a 29 year old c. Bowel cancer in a 32 year old 18. Pt with mononucleosis, which is atypical? a. Urinary epithelial with inclusions b. Atypical lymphocytes 19. Which of the following is not assoc with elevated alpha fetaprotein? a. HCC b. Crohns c. Ovarian/testicular cancer d. GIST 20. Psammoma bodies not seen in a) primary liver cancer b) thyroid cancer c) meningioma d) ovarian ca - serous 21.Newly diagnosed hemochromatosis. X-ray looking for involvement of a) liver b) heart 22. Which is false re melanoma a) dysplastic naevi b) radial growth is more imp for prognosis 23. Features of cystic lymphangioma do NOT include a. axillary or neck location b. child 18 months old c. large cystic spaces Page 67

24. LEAST FEMALE ASSOCIATED CONDITION a- Graves: b- MS c- PBC d- SLE 25. Which disease has the least female predominance ? A *Hashimotos. 10-20:1 B. *SLE C. Prim Biliary cirrhosis D. Another thyroid condition E. MS 26. WEBBED NECK AND FEMALE WITH THYROID DYSFUNCTION - Turners and Hashimotos - and hyperfunctioning adenoma - hypopit etc 27. BOWENS DISEASE - insitu BCC - metastatic adenoca 28. What is the most likely histological finding of a specimen from a patient with suspected Bowen disease ? A. * Squamous cell dysplasia B. * Epidermal spread of adenocarcinoma C. * Adenocarcinoma 29. Pt has candida septicaemia least likely ? A. Empyema B. Renal lesions C. mononuclear cells on LP E. Vegetations on tricuspid valve 30. most correct definition of hamartoma ? A. benign neoplasm composed of cells native to the organ B. benign neoplasm composed of cells foreign to the organ C. abnormal tissue in disorganized pattern composed of cells native. D. abnormal tissue in disorganized pattern composed of cells foreign 31. Disorganised growth, mature specialised cells or tissues indigenous to specific site a. Amyloidosis b. Hamartoma 32. Procedure planned, what is the least likely risk factor for DIC? a. prostatic Ca b. Giant Haemangioma c. Sarcoidosis d. Placental abruption

Page 68

33. Raised CEA least likely with a. smoking b. alcoholic cirrhosis c. Breast CA d. pancreatic CA e. RCC 34. Pt develop skin necrosis 4-12 hrs after contrast injection, same happens after being rechallenged this is a. type I allergic rx b. type II c. type Ill with arthus rxn d. not an allergic rx e. type IV 35. least likely tumor to find papillary histology a. breast CA b. Thyroid CA c. Craniopharyngioma (papillary and adamantinomatous) 36. Bordetella pertussis a) exotoxin b) cause laryngotracheobronchitis c) peripheral lymphocytosis d) membrane - di

GU
1. HUS - ? most likely a. following viral infection b. kidneys may appear normal acutely 2. Adult Haemolytic Uremic Syndrome 3. Haemolytic Uraemic Syndrome a. Due to infection with Rotavirus 4. Patient with previous Goodpastures, on dialysis. Gross hematuria and hypotensive. ?cause a. prostate vol 25cc b. renal cysts c. renal calculi d. amyloid 5. Patient with Goodpastures Syndrome and on longterm haemodialysis. Sudden onset of haematuria and death. Which is most significant on ultrasound a. multiple renal cysts b. renal calculi 6. Hypertensive patient most likely findings a. 2cm adrenal Carcinoma b. 10cm adenoma c. bilateral benign tumours Page 69

d. bilateral hyperplasia 7. Patient with hypertension and adrenal mass. What is true? a. Bilateral adrenal masses makes phoechromocytoma likely 8. Adrenal atrophy secondary to sustained HTN 9. Focal renal infarct, least likely: a) Atherosclerosis b) Mitral valve regurgitation c) PAN d) HUS) e) Post AMI f) Endocarditis 10. Regarding renal cell carcinoma. Which of the following is the LEAST CORRECT: a. Strong association with Von-Hippel-Lindau syndrome b. Clear cell carcinoma is the most common c. Increased risk of renal cell carcinoma in chronic renal failure d. Most frequent presentation is flank pain or discomfort e. Hereditary tumours are more frequently bilateral and multifocal 11.Fatty lesion next to effaced kidney. ?Liposarcoma Which of the following is most relevant? a. Tuberous sclerosis 12.Incidence of polycystic kidneys a. 1:1000 b. 1:10000 c. 1:100000 d. 1:10 Robbins Page 963 1 in 400 to 1000 13. Which of the following is not derived from renal tubules? a. papillary cell carcinoma b. clear cell carcinoma c. oncocytoma e. renal tubular adenoma f. angiomyolipoma Oncocytoma collecting ducts RCC tubulular epithelium Renal papillary adenoma - renal tubular epithelium 14. Renal disease, which is correct a. Interstitial cystitis/ Hunner Ulcer is associated with SLE b. Malakoplakia is metaplasia secondary to chronic infection c. 5% of bladder tumours are adenocarcinoma d. 5 year survival for grade I tumours is 75% e. sessile bladder tumour 1/3 of low grade, 80% of high grade TCC Robbins page 1028 15.Which is not a type of RCC a.Small cell Page 70

b.Clear cell c.Papillary d.Collecting duct e.Chromophobic 16. Oncocytoma a. a/c for 5% renal tumours b. benign 17. 62 year old man, large vascular mass found in kidney on CT scan a. Most likely an oncocytoma if it contains fat b. If it doesnt contain fat then it is likely to be an oncocytoma c. Probably an incidental finding of RCC d. Most likely to be a RCC 18. 6 week old child on ultrasound has enlarged kidneys with multiple 1 2mm cysts throughout the kidney. Also has moderate renal/mild hepatic impairment. Ultrasound of the parents and 2yr old asymptomatic sisters kidneys a. Parents normal, sister involved b. Father has a forme Fruste form of the condition c. Parents and sister normal d. (something about autosomal dominant form of the condition) 19. Which is false regarding nephroblastomatosis a. resolve by 1year of age b. precursor of wilms disease c. include nephrogenic rests and nephrobalstoma? In adult population d. renal hamartoma 20. 5 year survival for RCC with extension into the renal vein a) 90% a) 70% b) 30% Robbins page 1018. 15-20% 21. Wilms tumour, age at presentation a) 2-5 years b) c) 22. CEA is least likely to be associated with h) cirrhosis i) hepatitis j) smoking k) alcoholic cirrhosis l) breast cancer m) pancreatic cancer n) RCC 23. The features of analgesic nephropathy do not include a. Renal artery stenosis b. Renal calculi Page 71

c. Increased urinary Tamm-Horsfall protein. d. Nephritis e. TCC 24. Features of Clear Cell Renal Cell Ca include a. commonest at lower pole b. tan/white tumour macroscopically. 25. Wilms tumour overall survival ? a. 40-50 % b. 80-90% c. 5-year survival rate 90% long-term survival rates 26. Magnesium ammonium phosphate stones associated with: 27. Renal infarction most likely cause is a.PAN 28. Vascular 5cm renal mass found on investigation for RAS which was normal. ?Most likely a. incidental RCC b. RCC as cause for hypertension ? c. Oncocytoma d. Need to check for urinary VMA before biopsy e. TCC bladder 29. Cryptorchism a. Assoc with inguinal hernias b. Malignancy c. Infertility 30. Which of the following are all recognised bladder carcinoma morphological patterns: a. Papillary, ulcerative, diffuse, cystic, infiltrative b. Papillary, tubular, sessile, excavative c. Papillary, tubular, cystic, flat d. Flat, excavative, polypoid e. Flat, papillary-invasive, papilloma-papillary 31. The most likely explanation between magnesium ammonia phosphate stones and urinary tract infections is: a. Bacterial enzymes convert the natural urea to ammonia b. Inflammatory debris acts as a strong nidus for stones c. Non-degradable bacterial wall concentrates organic metals d. Bacterial plasmids promote heavy metal accumulation as a defence against leukocyte attack e. Acute inflammation lowers the pH <5.5 favouring crystal precipitation 32. Regarding prostatic carcinoma. Which of the following is the least correct: a. 70-80% of males between the ages of 70-80 have foci b. Histologically sheets or cords of cells in dense fibrous sroma c. 70% arise in the peripheral zone d. Local invasion into the seminal vesicles, bladder and ureter e. Carcinoma of the prostate is less common in Asia compared with Australia and New Zealand Page 72

33. Following NOT associated with increased risk of testicular malignancy a. Cryptorchidism b. Testicular feminisation c. Testicular microlithiasis d. Klinefelters syndrome e. TB of the testis 34. Regarding pheos a. T2 hyperintense b. Extra-adrenal pheos are less malignant than those associated with adrenal c. 2cm at Dx 35. With regards to germ cell tumours, which is false? a. Choriocarcinoma >5cm mass 36. Most likely explanation between MAG stone and UTI? a. Urea splitting organism b. Inflamm debris acting as a nidus for stones c. Non degradable bacterial wall 37. Which is not a sex cord stromal tumour? a.Yolk sac 38. BPH a)20% incidence male aged 40 b) arises in central zone c) corpora amylacea Ca within d) foci of squamous metaplasia and infarction 39. Prostatic carcinoma - which is false a. 70% males b/w 70-80 b. common in Asians c. 80% have foci of PIN d. Gleesons score correlates with well and poorly differentiated 40. Cryptorchidism a. Most commonly bilateral b. Is associated with infertility even after orchidopexy c. Is not associated with inguinal hernia as canal is not formed 41. 30 yr old with testicular mass which involves the entire testes and is uniformly bland in appearance and elevated BHCG a. choriocarcinoma b. 1/8 of seminomas secrete BHCG & should be considered c. Lymphoma d. Mixed yolk sac tumour 42. Man presents for biopsy of a solitary peripheral mass in the prostate with ipsilateral seminal vesicle enlargement, which is most correct? a. Prostate cancer doesnt occur in the periphery b. Multiple biopsies are indicated Page 73

c. Prostate cancer doesnt invade the seminal vesicles d. Unlikely to be prostate cancer if an African American 43. Which is not a morphological type of bladder cancer 44. Choriocarcinoma of the testis. What is the least likely? a) usually involves the whole testis with capsular invasion 45. Features of Bladder tumours do NOT include a. 70% solitary b. low grade lesions tend to be flat c. Transitional cell in 95% 46. Elderly man with polycythaemia. Non-associated finding on CT is a. Emphysema b. Renal mass c. Chronic pancreatitis d. Splenomegaly

Head and Neck


1. 55yo man - Oncocytes in biopsy specimen from solitary parotid cyst a. benign pleomorphic adenoma b. warthins c. HIV d. Type 1 branchial cleft cyst e. Mucoepidermoid Robbins page 793 2. Solitary lytic mass parotid, FNA polygonal cells, lymphocytes a. HIV lymphoepithelial cyst b. Warthins tumour c. Pleomorphic adenoma d. Mucoepidermoid carcinoma StatDx Warthins 3. 76 year old male for radiotherapy of thyroid lesion. Most likely? a. Anaplastic b. Follicular c. Medullary d. Papillary e. Lymphoma 4. Chronic otits media / mastoiditis, renal lesion, hilar mass. FNA shows non specific granuloma, best explained by a. wegner granulomatosis b. disseminated TB c. disseminated sarcoid d. polyarteritis nodosa e. goodpastures disease

Page 74

5. Difference between hamartoma and choristoma a. Same thing hamartoma an American term, choristoma European b. Choristoma has both mesenchymal and epithelial components Choristom = A collection of normal cells in an abnormal location. Example: adrenal choristoma Hamartoma = a mass of disorganized but mature specialized cells or tissue indigenous to the particular site, 6. Nasopharyngeal carcinoma - what is false? a. carcinoma at young age in african population b. therapy is surgery and radiotherapy c. 80% 5 year survival rate NPC XRT is standard Rx, usually not resectable 50-70% 3yr survival 7. Laryngeal cancer ? a. Associated with alcohol, smoking and irradiation b. Polyps and nodules are associated with increase risk of cancer c. Usually arises from false cords d. Polyps and nodules usually involve false cords 8. Paragangliomas a. paraganglioma, chemodectoma and carotid body tumours can be used interchangeably b. often adherent to vessels resulting in incomplete excision and recurrence of 10% 9. Least likely to cause hyperthyroidism. a. Graves disease b. Toxic multinodular goitre c. Toxic adenoma d. Hashimotos thyroiditis e. Reidels Thyroiditis f. Subacute g. Viral thyroiditis 10. Hashimotos FNA findings a. lymphoid infiltrate, increased Hurthle cells 11. DeQuervain macroscopic appearance a. Diffuse smooth symmetrical goitre b. Multiple nodules c. Bilateral or unilateral enlarged lobes d. Only microscopic changes Robbins page 1171 12. A 76 year old man, 5 cm thyroid mass, previously 2cm thyroid mass on imaging only 2 months earlier, when he had staging for an apparent Dukes B carcinoma of the caecum. The thyroid mass is most likely to be; a) Papillary carcinoma of the thyroid b) Follicular carcinoma of the thyroid c) Anaplastic thyroid carcinoma d) Primary thyroid lymphoma e) Metastatic colon carcinoma Page 75

13. 70 year old man with a 5 year history of a slowly enlarging, painless parotid mass which has markedly increased in size over the last 3 mths. The most likely cause is: a) Pleomorphic adenoma with malignant transformation to adenocarcinoma b) Infected first branchial cleft cyst c) Warthins Tumour with cystic change d) Haemorrhage into a pleomorphic adenoma e) Erosion and fistula.. 14. Definition of Thymoma a) Any benign thymic tumour b) Benign tumour of epithelial cells c) Any tumour of thymic epithelial cells d) Any tumour of thymus e) Benign tumours of epithelial and mesenchymal origin 15. Comparing Warthins tumour v. Pleomorphic adenoma of salivary glands a. Warthins is more likely bilateral. 16. LEAST LIKELY DIAGNOSIS OF NECK LUMP IN >60 Y.O. PATIENT WITH SCANT SQUAMOUS CELLS ON BIOPSY - metastatic SCC - adenoidcystic ca - mucoepidermoid - pleomorphic adenoma - branchial cyst 17. Least likely RF for thyroid ca a. Gardners b. Father with thyroid CA c. Mother with thyroid CA d. Hx of Hashimotos 18. Commonest cause of hyperthyroidism ? A. Graves. 19. ?if q directly about Hashimotos a. Patients with Hashimoto disease are at increased risk for the development of B-cell lymphomas. 20. Nasal polyps 21. Laryngeal Ca ? 22. Cholesteatoma ; which is most correct ? A. Stratified squamous epithelium with keratinous material B. contains cholesterol crystals 23. Hypoparathyroidism a. Associated with adrenal. 24. Retinoblastoma, which is the least likely Page 76

a) very radiosensitive. Excellent prognosis even if it extends retro orbitally b) carriers of RB gene have a 90% risk c) can get extra ocular retinoblastomas d) radiation induced sarcomas in 20% 25. Laryngeal carcinoma a) associated with asbestos exposure and smoking 26. Laryngeal Tumour most correct statement a) singers nodules on false cords b) carcinoma on false cords c) carcinoma above true cord 50. FNA of the thyroid shows scant cells and Psammoma bodies a) papillary ca

O&G
1. What serological marker not useful a. CA 125 - endometrial ca b. Alfa-fetoprotein cirrhosis c. Calcitonin - thyroid mass d. Beta HCG ??? gynae tumour 2. 34/40 fetus with large mass protruding posteriorly from sacrum. No evidence of Chiari malformation. Most likely is a. benign sacrococcygeal teratoma b. malignant sacrococcygeal teratoma c. congenital neuroblastoma d. imperforate cloacal membrane e. mature ganglioneuroma 3. Bilat ovarian cysts, internal septations, nodularity, calcifications a. Statistically mucinous adenocarcinoma most likely b. Morphologically serous and mucinous cant be differentiated c. calcification indicates psammoma bodies and therefore mucinous cystadenocarcinoma d. serous cystadenocarcinoma 4. Hematometra with cervical lesion involving upper vagina a. Stage II b. Stage III c. Vaginal primary 5. 40yr old female with hematometra & mass which involves the cervix and through to the bladder mucosa. Which is the most likely? a. Stage lic cervical cancer b. Stage IV cervical cancer c. Bladder cancer d. Vaginal Cancer 6. Thickened endometrial mass shows areas of osteoid, cartilage and muscle on biopsy. Most likely Page 77

a. hamartoma b. Metastatic immature teratoma c. endometrial carcinosarcoma 7. Patient with endometrial biopsy showing ?bone/cartilage etc. what is likely? a. primary uterine teratoma b. metastatic teratoma c. other teratoma option ?benign teratoma etc d. endometrial carcinosarcoma 8. Least likely to have 3 cells of origin a. Neonate with sacrococcygeal teratoma b. Female with benign teratoma c. Testicular tumour 9. Solid mass on ovary/ovarian tumour-not cystic/least likely to be cystic a. Brenner b. Dermoid c. Mucinous cystic adenoma d. Serous e. Endometroid 10. What is atypical in endometrial carcinoma ? a. Focal mass b. Diffuse thickening of uterus c. Tubal to peritoneal spread d. If SCC then it is likely to be cervical carcinoma spreading to the uterus Robbins page 1088 top left corner 20% of endometroid have some squamous diff; 20% of endometrial carcinoma resemble serous ovarian tumors and have a propensity for tuboperitoneal spread 11. What is (not?) typical of endometrial carcinoma? a) focal mass b) diffuse thickening of the uterus a) tubal to peritoneal spread c) if SCC then it is likely to be cervical carcinoma spreading to the uterus 12. Endometrial abnormality incorrect a. Polyps most commonly sessile b. Cancer may present with discrete mass c. Cancer may present with diffuse endometrial thickening d. Hopefully an incorrect option included 13. Twins incorrect a. Dichorionic cannot be monozygotic 14. Which of the following is not a congenital CNS infection a. chickenpox b. rubella c. CMV d. Toxoplasmosis e. Herpes Page 78

15. Regarding endometrial polyps which is true a. Best seen in proliferative phase b. Differentiating benign submucosal fibroid and endometrial polyp by US is possible c. Differentiation between fibroid and endometrial polyps possible on T2W d. When associated with endometrial fluid as indication of malignancy 16. 38yo with bilateral enlarged ovaries and marked endometrial thickening. ?cause b) Polycystic ovaries c) Granulosa theca ovarian tumour d) Endometrial carcinoma with bilateral ovarian metastases endometrial thickening in 30-40%: StatDx 17. Who is at the greatest risk of hydatiform mole a) Israel b) Sweden c) Netherlands d) USA e) Indonesia. 18. Diagnosis of trophoblastic disease is dependent on a) curettage b) histopathology absence of syncitiotrophoblasts 19. Sex cord tumor of the ovary include all the following except 20. Regarding cervical carcinoma -Carcinosarcoma is an atypical form -CIN III has koilocytes -CIN I is like condyloma accuminata 21. A fifty year old female develops a colovaginal fistula. Ten years earlier she had treatment for choriocarcinoma and colonoscopy six months ago was normal. Most likely cause is -Radiotherapy change -Ca Colon -Crohns -Diverticular abscess -Ca Bladder 22. Cystadenocarcinoma. Various questions on bilaterality and relative incidence of serous v mucinous. 23. A 16yr old female 140cm in height and weight 57kg has primary amenorrhoea. Most likely cause -Testicular feminisation -Prolactinoma -Anorexia -Polycystic ovaries -Turners syndrome 24. Pseudomyxoma peritoneii in perimenopausal female best explained by -Ovarian mucinous cystadenocarcinoma Page 79

-Ca Stomach -Mucocoele of appendix and rupture 25. Chorionic villus sampling at 11/40 reveals trisomy 7 (seven). Mother decides to continue pregnancy and routine USS at 18/40 shows viable fetus with normal morphology. Most likely explanation is -Trophoblast margin sampled -Mosaicism confined to the placenta -Trisomy 7 has bowel and palatal defects not visible on US -Trisomy 7 associated with hydrops and fetal death in third trimester 26. Female with 5cm solid ovarian lesion and endometrial thickening (? if it stated thickness) A. Granulosa Theca cell B. Chorio C. Serous cystadenoca D. Mucinous CystAdenoCa 27. Which of following ovarian tumours is of surface epithelial origin ? A. Teratoma B. Dysgerminoma C. Endometrioid D. Chorio E. Thecoma 28. Ca125 tumour marker is found in ? A. Ovarian carcinomas B. ovarian carcinomas and benign tumours/other ovarian pathology (?) 29. ENDOMETRIAL CANCER RISK FACORS - OBESITY - HTN - D.M. - Excess progesterone 30. COMPLETE MOLE MOST LIKELY - 46XX male X only - 46XX male only genetic material - 46XX or XY - triploidy 31. CA 125 - look for adenoca - look for primary ovarian cancer - look for primary colon cancer 32. Incidence of Ectopic pregnancy a. 1 in 150 b. 1 in 300 c. 1 in 1500 d. 1 in 3000 e. 1 in 5000

Page 80

33. CA 125 raised in a. Ca ovary b. Ca colon c. Ca colon and ovary d. Ca pancreas 34. Regarding cervical carcinoma -Carcinosarcoma is an atypical form -CIN III has koilocytes -CIN I is like condyloma accuminata 35. Chorionic villus sampling at 11/40 reveals trisomy 7 (seven). Mother decides to continue pregnancy and routine USS at 18/40 shows viable fetus with normal morphology. Most likely explanation is -Trophoblast margin sampled -Mosaicism confined to the placenta -Trisomy 7 has bowel and palatal defects not visible on US -Trisomy 7 associated with hydrops and fetal death in third trimester 36. Ovarian tumour with solid and cystic components + papillary folds + calcification a. serous cystadenomaICA b. mucinous cystadenoma/CA c. brenners 37. Ovarian endometriod Ca surprise Dx, see what on TVS? a. 15-30% have endometrial Ca b. 10% Cervical stenosis 38. Distribution of endometriosis which order a. Various combinations of pelvic floor, surgical scar?, laporoscopy scar, chest/lung, Pouch of Douglas 39. Adenomyosis a. Doesnt exist b. Down growth of endometrium c. Intra muscular endometrium same pathology

CVS
1. Which of the following statements with regards to fatty plaques is LEAST CORRECT? a. Moreover, they frequently affect individuals in geographic locales and populations in which atherosclerotic plaque is uncommon. b. ?% of kids that have them c. ?site d. bld vess and heart pg 503 blue box know well 2. Regarding polyarteritis nodosa. Which of the following is LEAST CORRECT: a. The kidneys are most commonly involved in autopsy b. Aneurysms are seen at angiography in 50% Page 81

c. 70% are seropositive for Hepatitis B surface antigen d. Churg Strauss syndrome frequently involves pulmonary and splenic vessels e. Serum anti-neutrophil antibody titres correlate with disease activity 3. Which of the following would be an UNEXPECTED finding following coronary angioplasty? a. Luminal expansion b. Plaque rupture c. Thickened intact intima d. Medial dissection e. Proliferative restenosis in 30-50% of patients at 6 months 4. Which of the following conditions IS TRUE with regards to hypertension? a. 5-10% of dissections have no obvious intimal tear. 5. Regarding aortic coarctation, which of the following IS NOT a well recognised association: a. PDA b. Ventriculoseptal defect c. Dural AVM d. Berry aneurysms of the circle of Willis e. Bicuspid valve 6. Regarding aortic dissection. Which of the following is the MOST CORRECT: a. 5-10% do not have an obvious intimal tear b. Hypertension is an antecedent in 50% of cases c. Haemorrhage typically occurs between the inner and middle thirds of the media d. Elastic fragmentation of the media is rare at autopsy in patients free of dissection e. Intimal tear is found in the proximal descending thoracic aorta in 90% of cases 7. Wegeners granuloma, commonest site A. Lung B. Paranasal sinus C. Nasopharynx D. Kidneys E. Skin Lung Robbins. PN sinus - Webb 8. Which of the following conditions is not a cause (or association) of dissection A. Hypertension B. Arteriosclerosis C. Pregnancy D. Marfans syndrome 9. Kawasaki disease A. Mostly occur in children younger than 2 years of age B. Is self limited C. >20% develop CVS sequale 10. A myocardial infarct in a post-mortem specimen has a white centre and red margins. The likely age of the infarct is -24-36 hours -3-8 days -14-21 days Page 82

11. What is most important factor in myocardial infarction? A. Size B. Site C. Transmural versus subendocardial D. Involvement of right ventricle wall 12. Sudden cardiac death is associated with the following except -Mitral valve prolapse -Mitral stenosis -Aortic stenosis -HOCM -Pulmonary Hypertension -myocarditis -coronary art -structural abn 13. Kawasakis disease is -A self-limiting disorder with less than 1% death rate -Associated with a 20% death rate -Characterised by recurrent attacks 14. Fifty year old female presents with stenoses of the proximal carotid arteries and subclavian arteries. Most likely diagnosis is -Takayasus disease -FMH -Atherosclerosis 15. A myocardial infarct in a post-mortem specimen has a white centre and red margins. The likely age of the infarct is -24-36 hours -3-8 days -14-21 days 16. ASD septum secondum the most correct statement is: a.An isolated finding b. close to A Vjunction c. 1eft ventricular fai]ure 17. Left ventricular changes secondary to Hypertension include: 18. Regarding infective endocarditis, which of the following is the LEAST CORRECT: f. Right sided heart valves in 50% of narcotic related cases g. Diffuse glomerulonephritis seen in 50% of untreated cases h. Positive blood cultures are seen in 85-90% of cases i. Strep pneumonia is the leading cause of acute endocarditis j. With mechanical prostheses infections are usually located on the margin of the sewing ring 19. Atrial Myxoma a. can prolapse into ventricle b. pedunculated c. gelatinous papillary Page 83

20. Patient with AMI, develops chest pain 5 days later, least likely cause is k. 2nd AMI l. Dresslers syndrome m. Bronchopneumonia n. Pulmonary Embolism o. Myocardial rupture 21. Aortic dissection. i. Can occur without an intimal flap ii. Longitudinal iii. 30-50cm long iv. aortic valve prevents involvement coronary arteries 22. Regarding cardiomyopathies, which of the following is the most correct: i. Can involve the right ventricle ii. Autosomal recessive cases are seen in over 50% of cases of hypertrophic CMP iii. Atria never involved in hypertrophic CMP iv. Endomyocardial fibrosis is mainly a disease of children and young adults in South America v. dilation of the heart in peripartum CMP is irreversible 23. i. ii. iii. 24. i. 25. a) Mitral ring calcification Soft hard, MVR & conduction defects normal variant ASD secundum, ? true 60% of ASDs usually isolated Rheumatic valve, ? which affected Mitral and aortic

26. Which doesnt cause segmental wall hypomotion: a) endocardial fibroelastosis b) infarct acute c) infarct old d) hibernation e) stunned myocardium f) amyloid 27. Giant cell arteritis - which is true a. involves veins b. diffuse/ continous involvement of vessel c. temporal arteries only d. granulomas and langerhan cells e. ?giant cells and langerhan cells f. <50 years and males 28. Type B dissection defined by a. distal to subclavian artery b. distal to ligamentum arteriosum C. no involvement of ascending arch Page 84

d.

entire aorta

29. question on Takayasu. Responses focused on a. female or male 8 :1 b. age 12-66 C. histological findings granulomatous inflam 30. Went to a conference and a paper discussed reperfusion injury with MI (?transient). This paper was actually referring to a. stunned myocardium b. hibernating myocardium c. ischaemic penumbra d. free radicals 31. a. b. c. d. Heart. Day 5 AMI for CT, least likely Pericarditis ?extension of infarct ?PE LV aneurysm

32. Heart. 45% reversible defect, 70% Ejection fraction, what does it mean for prognosis a. Decrease EF to 30% if another infarct b. Statistically normal finding post AMI 33. Aortitis 40?yo clinicans query syphilitic aortitis. Not true a. Endothelial abN on Angio b. Invol vaso vasorum with infiltrates c. Is tertiary manifestation d. No because is fibrous no dilatation e. Cannot access degree of ?? 34. Vasculitis - PAN a. blind b. pulseless disease c. 5 yo d. invol kidney 35. Causes of pericarditis least likely a. Tb b. chronic renal failure c. cocksackie virus d. post MI e. Cryptococcus 36.Type B dissection - definition a. Involves asc and desc aorta b. Arises distal to left SCA c. Arises distal to left CCA d. Arises distal to ligamentum arteriosum e. Involves desc thoracic aorta 37. Suspected bacterial endocarditis - which is least likely? Page 85

a. b. c. d. e.

Candida repeated negative blood cultures liver infarct hematuria MCA aneurysm

38. An article on myocardial infarction talks of reperfusion injury. This is best described by? a. A special form of damage, probably free radical mediated b. The injury that persists after revascularisation of an infarct c. haemorrhage into an area of ischemic necrosis d. resolution of early reversible ischemic cellular changes e. myocardium viable but shows a transitory depression of function (stunned myocardium) 39. Cardiac echo with focal abnormal ventricular wall motion - least likely cause a. AMI b. old infarct c. stunned myocardium d. hibernating myocardium e. endomyocardial fibroelastosis 40. What is least likely finding on CXR if pt has restrictive cardiomyopathy ? A. Bronchiectasis B. Bilat hilar lyphadenopathy C. Basal bullous disease D. ? fibrosis ?scleroderma 41. Bicuspid aortic valve ? A. occurs in 1% of the population 42. Aortic Dissection classification of two patients in which they describe the origin and termination of the dissection ? A-E. different combinations of Stanford and DeBakey class 43. Patient A has dissection involving ascending aorta, Patient B has dissection involving ascending aorta A. Both have type A B. Both have type B C. Patient A has type A and patient B has type B D. Patient A has DeBakey III and patient B has type B E. Patient A has DeBakey I and patient B has DeBakey III 44. Man with right sided heart failure, least likely finding is A. Liebman Sachs endocarditis B. Arrythmogenic right ventricular cardiomyopathy C. Asbestosis D. Ischaemia involving only right ventricle E. Ischaemia involving right ventricle and posterior wall of left ventricle 45. Patient with restrictive cardiomyopathy, least likely finding on chest CT is A. Basal bullous change B. Radiotherapy changes C. Bilateral hilar lymphadenopathy Page 86

D. E.

Multiple pulmonary masses Bilateral bronchiectasis

46. In hypertensive heart disease, which is least true A. Circumferential enlargement of left ventricle B. Mild ventricular dilatation, classically accompanies cardiac enlargement C. Mild diastolic filling impairment D. Ventricular wall thickness >2cm E. Increase in muscle mass more than cardiac size 47. Complications of mitral valve prolapse, which is false A. Aortic regurgitation secondary to annular dilatation B. Infective endocarditis C. Stroke D. Sudden death 48. Autopsy on 40 year old female shows abnormal aortic valve with 2 leaflets, one of which is large with median raphe, which is true A. Bicuspid aortic valve occurs in 0.1% of population B. Bicuspid aortic valve occurs in 1% of population C. Patient has rheumatic heart disease D. Patient has calcific aortic stenosis E. Patient has Marfans 49. In Secundum ASD, which is most true A. Accounts for 60-70% of ASDs B. Occurs in atrioventricular valve C. Usually isolated anomaly D. Associated with left ventricular enlargement E. Present in neonatal period 50. Short female patient with webbed neck and previous coarctation repair presents for thyroid ultrasound-most likely A. Turners with thyroid adenoma B. Turners with Hashimotos Thyroiditis C. Familial medullary carcinoma D. MEN II E. Achondroplasia 51. 40 year old female with stroke, underlying cause least likely is A. Atherosclerosis B. Dissection C. Coarctation of aorta D. Giant cell arteritis E. Mitral valve prolapse 52. RHEUMATIC HEART LESIONS secondary to (most correct) A. GROUP B haemolytic staph cross reactivity B. Cross reactivity with organism capsule C. Direct toxic effect on valve 53. SYSTEMIC HYPERTENSION AND HEART CHANGES most correct Page 87

A. B. C. D.

increase mass > size symmetrical hypertrophy hypertrophy and mild dilatation may be associated with diastolic dysfunction

54. RESTRICTIVE CARDIOMYOPATHY FINDINGS LEAST LIKELY ON CXR A. hilar adenopathy B. multiple pulmonary nodules C. basal emphysema 55. SYSTEMIC SCLEROSIS AND HYPERTENSION - ? MOST LIKELY CAUSE A. intimal proliferation of interlobar arteries B. pipestem fibrosis of adventitia of proximal renal aa C. malignant HTN secondary to accelerated atherosclerosis 56. Most likely cause of hypertension in patient with systemic sclerosis ? 57. Large exophytic lesion on cardiac valve ... most likely scenario ? A. Prescence of fibrinous pericarditis and ?pleural/pericardial effusion B. Splenic lesions makes candida most likely 58. Bicuspid aortic valve ? A. occurs in 1% of the population 59. 40 yo woman with enlarged pulmonary arteries, and pathologically a web like network of capillary formations within the vessel wall and lumen, is most likely secondary to ? A. primary pulmonary hypertension B. recurrent pulmonary emboli 60. Least likely association with MV prolapse ? A. AV pathology 61. The following are associated with aortic coartation except: a. Coarctation syndrome: triad of coarctation, PDA, VSD b. In Turner syndrome, most common cardiac abnormality c. Bicuspid aortic valve, 50% d. Circle of Willis aneurysms e. Intracardiac defects; occur in 20% with infantile type 62. Regarding TAPVC which is false:a. Supracardiac TAPVC is the most common type; infrequently associated with obstruction. b. Infracardiac TAPVC connection comprise 15%; majority are obstructed c. Snowman heart (figure-of-eight heart) in supracardiac type d. Snowman configuration not seen with other types e. Is an acyanotic lesion 63. ASD the least correct statement is a. most common cause of CHD diagnosed in adults b. most common type is ostium secundum type c. ostium primum defect associated with mitral & tricuspid valve defect d. dilatation of the pulm arteries e. characteristic feature is dilatation of LV/LA Page 88

64. In contrast to preductal type, postductal coarctation is more likely to a. Occur in adults b. Hypertension of the upper limbs c. LV hypertrophy d. Selective cyanosis of the lower limbs e. Closing of the ductus arteriosus 65. 15 yo presents with fevers, rash, polyarthritis and diastolic murmur. Which histologic finding is least likely a. Fibrinoid necrosis with inflamatory cells b. Anitschkow cells c. MacCallum plaques d. Erythema nodusum e. Bread & butter pericarditis 66. CXR findings in EISENMENGER SYNDROME are as follows except a. pronounced dilatation of central pulmonary arteries b. redistribution of pulmonary veins c. pruning of peripheral pulmonary arteries d. enlargement of RV e. LA + LV return to normal size 67. ASD which is least correct a. Most common congenital cardiac anomaly. b. Ostium secundum type: most common, approx 60% c. Ostium secundum type as with Downs syndrome d. Ostium primum ASD may be part of the ECD syndromes e. Sinus venosus defect always associated with PAPVC anomalous venous return 68. Restrictive cardiomyopathy least likely CXR findings a. Pericardial calcification b. Pleural effusion c. Pruning of peripheral pulmonary arteries d. PVH e. Dilated SVC and azygos 69. ASD the least likely CXR finding: a. loss of visualization of SVC (= clockwise rotation of heart due to RV hypertrophy) b. small appearing aorta with normal aortic knob c. enlargement of pulmonary trunk + arteries d. RV enlargement e. enlargement of LV 70. The least accurate statement regarding VENTRICULAR SEPTAL DEFECT is a. most common CHD overall b. isolated in 20% c. acyanotic L-to-R shunt d. right aortic arch in 50% e. MEMBRANOUS type in 75%

Page 89

71. The most incorrect statement is a. Fatty streaks seen in all children > 10 yrs and may cause atheromatous plaques b. Monckebergs a/s rim calcification in large-medium sized arteries but do not narrow the lumen c. Both essential & secondary HTN may be malignant d. Hyaline a/s is common in diabetes and elderly even in normotensives e. Hyperplastic a/s demonstrates onion skin wall thickening and is characteristic of malignant HTN 72. The following differentiates microscopic polyangiitis from PAN a. Necrotising glomerulonephritis is common b. Pulmonary capillaritis is common c. Muscular medium sized arteries are spared d. Commonly involves post capillary venules e. p-ANCA in 75% 73. The least likely associations a. Hep B & PAN b. cANCA & Wegeners c. PAN & cANCA d. pANCA & microscopic polyangiitis e. antiendothelial abs & SLE 74. LEAST likely association of Abdominal Aortic Aneurysm: a. isolated iliac + femoral artery aneurysm b. stenosis / occlusion of celiac trunk / SMA c. occlusion of inferior mesenteric artery d. visceral + renal artery aneurysm e. occlusion of lumbar arteries 75. BUERGER DISEASE the incorrect statement is a. multinucleated giant cells within microabscesses is pathognomonic b. associated with cigarette smoking in 95% c. starts in palmar + plantar vessels with proximal progression d. usually continuous involvement with tapered narrowing of arteries e. absence of generalized arteriosclerosis / arterial calcifications 76. Syphilitic Aneurysm, incorrect a. chronic inflammation of aortic adventitia + media beginning at vasa vasorum b. asymmetric enlargement of aortic sinuses c. fine pencil-like calcifications of intima in 20% in ascending aorta d. involves the ascending aorta in 75% e. Massively enlarged heart / LV 77. TAKAYASU ARTERITIS the least correct statement is a. granulomatous inflammation of unknown pathogenesis b. limited to persons usually <50 years of age c. only form of aortitis that produces stenosis / occlusion of the aorta d. Morphologically indistinguishable from temporal arteritis e. Associated with polymyalgia rheumatica in 50% 78. HENOCH-SCHNLEIN PURPURA, incorrect Page 90

a. b. c. d. e.

most common systemic allergic vasculitis in children precipitated by bacterial / viral infection, allergies, insect sting,or drugs acute febrile multisystem vasculitis with a predilection for the coronary arteries deposition of IgA-dominant immune complexes in venules, capillaries, and arterioles renal insufficiency in upto 20%, with end-stage renal disease in 5%

79. The least likely association of isolated hepatic angiosarcoma is a. Stewart-Treves syndrome b. thorotrast c. polyvinyl chloride d. hemochromatosis e. von Recklinghausen disease

Page 91

You might also like